prelims academy for civil services press · 2.13.1 mission rakshagyan shakti 2.14 national food...

50
GOVT. SCHEMES AND REPORTS Academy for Civil Services Powered by: PRELIMS PRESS

Upload: others

Post on 20-Aug-2020

1 views

Category:

Documents


0 download

TRANSCRIPT

Page 1: PRELIMS Academy for Civil Services PRESS · 2.13.1 Mission RakshaGyan Shakti 2.14 National Food Security Act (NFSA) 2.15 Ministry of Rural Development 2.15.1 Mission Antyodaya 2.15.2

GOVT. SCHEMESAND REPORTS

Academy for Civil Services

Powered by:

PRELIMS

PRESS

Page 2: PRELIMS Academy for Civil Services PRESS · 2.13.1 Mission RakshaGyan Shakti 2.14 National Food Security Act (NFSA) 2.15 Ministry of Rural Development 2.15.1 Mission Antyodaya 2.15.2

https://t.me/joinchat/AAAAAFYrI5kpQsEAKqmo-A

Page | 1

2.1 Ministry of Power

2.1.1 DeenDayalUpadhyaya Gram

JyotiYojana' (DDUGJY)

2.1.2 Saubhagya – ‘Pradhan Mantri Sahaj Bijli

Har Ghar Yojana’

2.2 Ministry of Labour and Employment

2.2.1 PM Rojgar ProtsahanYojana

2.2.2 Model Welfare Scheme for Building and

other Construction Workers (BOCW)

2.3 Ministry of Chemicals and Fertilizers

2.3.1 Nutrient based Subsidy Scheme

2.3.2 Pradhan Mantri Bhartiya Janaushadhi

Pariyojana

2.4 Ministry of Skill Development and

Entrepreneurship

2.4.1 National Apprentice Promotion Scheme

2.4.2 Pradhan Mantri Kausal Vikas Yojana

2.5 Ministry of Statistics and Programme

Implementation

2.5.1 Members of Parliament Local Area

Development Scheme(MPLADS)

2.6 Ministry of External Affairs

2.6.1 Know India Programme

2.6.2. SAMEEP

2.7 Ministry of Water Resources, River

Development and Ganga Rejuvenation

2.7.1 Namami Gange Programme

2.7.2 Atal Bhujal Yojana (ABHY)

2.8 Ministry of Culture

2.8.1 Seva Bhoj Yojana

2.8.2 Project Mausam

2.9 The Ministry of New and Renewable

Energy

2.9.1 Kusum (Kisan Urja Suraksha Evam

Utthaan Mahaabhiyan) Scheme

2.10 Ministry of Commerce and Industry

2.10.1 ReUnite

2.10.2 Revenue Insurance Scheme for

Plantation Crops

2.11 Ministry of Heavy Industries and Public

Enterprises

2.11.1 FAME-India Scheme

2.12 PM UjalaYojana

2.13 Ministry of Defence

2.13.1 Mission RakshaGyan Shakti

2.14 National Food Security Act (NFSA)

2.15 Ministry of Rural Development

2.15.1 Mission Antyodaya

2.15.2 Gram Swaraj Abhiyan

2.15.3 Shyama Prasad MukherjiRurban

Mission

2.15.4 Deendayal Upadhyaya Grameen

Kaushalya Yyojana (DDU-GKY)

2.15.5 Pradhan Mantri AwasYojana (Rural)

2.15.6 Deendayal Antyodaya Yojana –

National Rural Livelihood Mission (DAY-

NRLM)

2.16 Ministry of Social Justice and

Empowerment

2.16.1 Rashtriya Vayoshri Yojana (RVY)

2.17 Ministry of Micro, Small and Medium

Enterprises (MSME)

2.17.1 UdyamShakhi Portal

2.17.2 Prime Minister's Employment

Generation Programme (PMEGP)

2.18 Ministry of Petroleum and Natural Gas

2.18.1 UjjwalaProgramme

2.18.2 Sustainable Alternative Towards

Affordable Transportation (SATAT)

2.18.3 LPG Panchayat

2.19 Ministry of Finance

2.19.1 Pradhan Mantri Vaya Vandana Yojana

(PMVVY)

2.19.2 Atal Pension Yojana

2.19.3 PradhanMantri Jan DhanYojana

PradhanMantri Mudra Yojana

2.20 Ministry of Minority Affairs

Page 3: PRELIMS Academy for Civil Services PRESS · 2.13.1 Mission RakshaGyan Shakti 2.14 National Food Security Act (NFSA) 2.15 Ministry of Rural Development 2.15.1 Mission Antyodaya 2.15.2

https://t.me/joinchat/AAAAAFYrI5kpQsEAKqmo-A

Page | 2

2.20.1 PradhanMantri Jan VikasKaryakram

(PMJVK)

2.20.2 JiyoParsi Scheme

2.21 Ministry of Women and Child

Development

2.21.1 POSHAN Abhiyaan

2.21.2 Mahila Shakti Kendra Scheme

2.21.3 SHe box

2.21.4 SwadharGreh Scheme

2.21.5 BetiBachaoBetiPadhao(BBBP)

2.21.6 PradhanMantriMatruVandanaYojana

(PMMVY)

2.22 Ministry of IT and Electronics

2.22.1 Project Stree Swabhimaan

2.23 Ministry of Human Resource

Development

2.23.1 Rashtriya Avishkar Abhiyan

2.23.2 Unnat Bharat Abhiyan 2.0

2.23.4 Sustainable Action for 2

2.23.5 Transforming Human Capital in

Education

2.23.6 (SATH-E) Programme

2.23.7 Samagra Shiksha Scheme

2.24 Ministry of Tribal Affairs

2.24.1 Van Dhan Scheme

2.25 PM-AASHA

2.26 Ministry of Health and Family Welfare

2.26.1 NikshayPoshanAbhiyan

2.26.2 Pradhan Mantri Swasthya Suraksha

Yojana (PMSSY)

2.26.3 Ayushman Bharat Health Scheme

2.26.4 LaqshyaProgramme

2.26.5 Pradhan Mantri Surakshit Matritva

Abhiyan (PMSMA)

2.27Ministry of Agriculture and Farmers

Welfare

2.27.1Ensure Portal

2.27.2 Pradhan Mantri Fasal BimaYojna

(PMFBY)

2.27.3 KrishonnatiYojana

2.27.4 ParamparagatKrishiVikasYojana

2.27.5 National Bamboo Mission

2.27.6 Pradhan Mantri Krishi Sinchayee

Yojana (PMKSY)

2.28 Union Ministry of Housing and Urban

Affairs

2.28.1 PAiSA Portal

2.29 Ministry of Housing and Urban Poverty

Alleviation

2.29.1 PradhanMantriAwaasYojana (Urban)

2.30 Important State Government schemes:

2.30.1 NirmanKusumaProgramme

2.30.2 Sambal Scheme

2.30.3 Rythu Bandhu

2.30.4 Swayangsiddha Initiative

2.30.5 MukhyamantriYuvaNestham

2.30.6 Kanyashree Scheme

2.30.7 Krushak Assistance for Livelihood and

Income Augmentation (KALIA) scheme

2.30.8 SuryashaktiKisanYojana(SKY) Scheme

2.31 Others

2.31.1 Ujjwala Sanitary Napkins Initiative

2.32 Ashok Dalwai Committee: Doubling

agricultural income by 2022

2.33 Healthy states progressive India Report

Page 4: PRELIMS Academy for Civil Services PRESS · 2.13.1 Mission RakshaGyan Shakti 2.14 National Food Security Act (NFSA) 2.15 Ministry of Rural Development 2.15.1 Mission Antyodaya 2.15.2

https://t.me/joinchat/AAAAAFYrI5kpQsEAKqmo-A

Page | 3

2.1 Ministry of Power

2.1.1 Deen Dayal Upadhyaya Gram Jyoti Yojana (DDUGJY)

It was launched in 2015 by Ministry of Power with an objective to augment power supply to the rural

areas and to strengthen sub-transmission and distribution systems

Scheme:

It focuses on feeder separation (rural households & agricultural) and strengthening of sub-

transmission & distribution infrastructure including metering at all levels in rural areas

This will help in providing round the clock power to rural households and adequate power to

agricultural consumers

The earlier scheme for rural electrification viz. Rajiv Gandhi Grameen Vidyutikaran Yojana

(RGGVY) has been subsumed in the new scheme as its rural electrification component.

2.1.2 Saubhagya – Pradhan Mantri Sahaj Bijli Har Ghar Yojana

Key Points:

It was launched in September 2017 with

the objective to provide access to

electricity to all the remaining

households in the country. The scheme

primarily benefits rural areas, which

have vast majority of households

without power connections.

It is being funded to extent of 60% by

central grants, 30% by bank loans and

10% by states. Under it, free electricity

connections are provided to below

poverty line (BPL) households, while

other households have to pay 500

rupees for the connection.

Implementation-

To identify beneficiaries for free

electricity connections, the government

will use Socio Economic and Caste

Census (SECC) 2011 data.

The beneficiary household will get five

LED lights, one DC fan, one DC power

plug. It also includes the Repair and

Maintenance (R&M) for 5 years.

To ensure on-the-spot registration,

mobile applications will be used.

Gram Panchayat and public institutions

in rural areas will be authorised to carry

out billing and collection tasks.

There will be no subsidy component for

monthly electricity consumption.

8 States have achieved 100% household

electrification under Saubhagya namely Madhya

Pradesh, Tripura, Bihar, J&K, Mizoram, Sikkim,

Telangana and West Bengal. Total 15 States now

have 100 % household electrification.

2.2 Ministry of Labour and Employment

Page 5: PRELIMS Academy for Civil Services PRESS · 2.13.1 Mission RakshaGyan Shakti 2.14 National Food Security Act (NFSA) 2.15 Ministry of Rural Development 2.15.1 Mission Antyodaya 2.15.2

https://t.me/joinchat/AAAAAFYrI5kpQsEAKqmo-A

Page | 4

2.2.1 PM Rojgar ProtsahanYojana

The scheme is implemented by Ministry of Labour and Employment with an objective to promote

employment generation

Scheme:

Central Government will be paying the 8.33% EPS contribution of the employer for the new

employment.

New Changes:

The Central Government will now contribute Employer‟s full admissible contribution for first 3

years from date of registration of new employee for all sectors including existing beneficiaries for

their remaining period of 3 years.

All Industries registered with Employees‟ Provident Fund Organization (EPFO) can apply for

availing benefits under the scheme.

It is for new employees (joined on or after 1st April 2016) having new Universal Account Number

(UAN).

It is targeted for employees earning wages less than Rs. 15,000/- per month.

The scheme will be in operation for a period of 3 years and GOI will continue to pay and all new

eligible employees will be covered till 2019-20.

2.2.2 Model Welfare Scheme for Building and Other Construction Workers

(BOCW)

The Government has modified the Model Welfare

Scheme vide which it has been held by the

Government that the following social welfares

will hold precedence over the other existing

benefits:

Life and disability cover: State Welfare

Board should provide minimum

coverage of Rs. 4 Lakhs for accidental

deaths & Rs. 2 Lakhs for natural deaths

to the dependents of the deceased;

Health & Maternity cover: State Welfare

Boards need to ensure that the

„Ayushman Bharat‟ Scheme covers the

majority of the BOCW workers.

Education: Minimum financial

assistance for education of the wards of

the BOCW workers; the rates at which

the same should be provided is also laid

down under the Scheme.

Housing accommodation: State

Government to facilitate transit

accommodation/ labour shed cum night

shelter to BOCW workers.

Skill development: There should be

convergence of the skill development

activities of the BOCW Board with those

Page 6: PRELIMS Academy for Civil Services PRESS · 2.13.1 Mission RakshaGyan Shakti 2.14 National Food Security Act (NFSA) 2.15 Ministry of Rural Development 2.15.1 Mission Antyodaya 2.15.2

https://t.me/joinchat/AAAAAFYrI5kpQsEAKqmo-A

Page | 5

of the State Skill Development

Missions/Ministry of Skill Development

and Entrepreneurship and National Skill

Development Corporation.

Awareness Programme: Awareness

should be conducted at a grass root

level and not through advertisements

through newspapers or TV channels.

Pension: State Welfare Boards to

formulate pension schemes for BOCW

workers

2.3 Ministry of Chemicals and Fertilizers

2.3.1 Nutrient based Subsidy Scheme

It aims at ensuring balanced use of fertilizers, improving the agricultural productivity, promoting the

growth of the indigenous fertilizers industry and also reducing the burden of Subsidy. The Cabinet

Committee on Economic Affairs has approved the proposal of the Department of Fertilizers for the

continuation of Nutrient Based Subsidy (NBS) 12th Five Year plan till 2019-20.

Nutrient Based Subsidy (NBS) programme for fertilizers was initiated in the year 2010. Under the

scheme, a fixed amount of subsidy decided on an annual basis is provided on each grade of

subsidized Phosphatic and Potassic (P&K) fertilizers, except for Urea, based on the nutrient

content present in them.

It is largely for secondary nutrients like N, P, S and K and micronutrients which are very

important for crop growth and development. The scheme is being implemented by the Department

of Fertilizers.

2.3.2 Pradhan Mantri Bhartiya Janaushadhi Pariyojana

Pradhan Mantri Bhartiya Janaushadhi Pariyojana (PMBJP) is a campaign launched by the Department of

Pharmaceuticals to provide quality medicines at affordable prices to the masses. PMBJP stores have been

set up to provide generic drugs, which are available at lesser prices but are equivalent in quality and

efficacy as expensive branded drugs. It was launched by the Department of Pharmaceuticals in November

2008 under the name Jan Aushadi Campaign. Bureau of Pharma PSUs of India (BPPI) is the

implementation agency for PMBJP.

Vision

To bring down the healthcare budget of every citizen of India through providing Quality generic Medicines

at Affordable Prices.

Mission

Create awareness among the public regarding generic medicines.

Create demand for generic medicines through medical practitioners.

Page 7: PRELIMS Academy for Civil Services PRESS · 2.13.1 Mission RakshaGyan Shakti 2.14 National Food Security Act (NFSA) 2.15 Ministry of Rural Development 2.15.1 Mission Antyodaya 2.15.2

https://t.me/joinchat/AAAAAFYrI5kpQsEAKqmo-A

Page | 6

Create awareness through education and awareness program that high price need not be

synonymous with high quality.

Provide all the commonly used generic medicines covering all the therapeutic groups.

Provide all the related health care products too under the scheme.

Objective

Making quality medicines available at affordable prices for all, particularly the poor and disadvantaged,

through exclusive outlets "Jan Aushadhi Medical Store", so as to reduce out of pocket expenses in

healthcare.

2.4 Ministry of Skill Development and Entrepreneurship

2.4.1 National Apprentice Promotion Scheme

It is implemented by Ministry of Skill Development and Entrepreneurship with an objective to provide

apprenticeship training to over 50 lakh youngsters by 2019-20 in order to create more jobs.

Scheme:

Scheme will be implemented by Director General of Training (DGT)

Union Government will directly share 25% of the total stipend payable to an apprentice with

employers

Union Government will bear the 50% of the total expenditure incurred on providing basic training

to an apprentice.

The NAPS has been framed to meet objective of National Policy of Skill Development and

Entrepreneurship, 2015 which focuses on apprenticeship as one of the key components for

creating skilled manpower in India

2.4.2 Pradhan Mantri Kaushal Vikas Yojana

It is an outcome based skill certification scheme implemented by National Skill Development Corporation

(NSDC). The objective is to enable a large number of Indian youth to take up industry-relevant skill

training that will help them in securing a better livelihood. Individuals with prior learning experience or

skills will also be assessed and certified under Recognition of Prior Learning (RPL). Training and

Assessment fees are completely paid by the Government. The Various components are short term

training, recognition of prior learning, special projects, kaushal and rozgarmela, placement guidelines and

monitoring guidelines.

Page 8: PRELIMS Academy for Civil Services PRESS · 2.13.1 Mission RakshaGyan Shakti 2.14 National Food Security Act (NFSA) 2.15 Ministry of Rural Development 2.15.1 Mission Antyodaya 2.15.2

https://t.me/joinchat/AAAAAFYrI5kpQsEAKqmo-A

Page | 7

2.5 Ministry of Statistics and Programme Implementation

2.5.1 Members of Parliament Local Area Development Scheme(MPLADS)

It was launched in December, 1993 by Ministry

of Statistics and Programme Implementation. It

provides a mechanism for the Members of

Parliament to recommend works of

developmental nature for creation of durable

community assets and for provision of basic

facilities including community infrastructure,

based on locally felt needs.

Salient features:

MPLADS is a centrally-sponsored plan

scheme fully funded by the government

of India under which funds are released

in the form of grants in-aid directly to

the district authorities.

Works, developmental in nature, based

on locally felt needs and always

available for the use of the public at

large, are eligible under the scheme.

Preference under the scheme is given to

works relating to national priorities,

such as provision of drinking water,

public health, education, sanitation,

roads, etc.

The funds released under the scheme

are non-lapsable. Funds not released in

a particular year are carried forward to

the subsequent years, subject to

eligibility.

The MPs have a recommendatory role

under the scheme. They recommend

their choice of works to the concerned

district authorities who implement these

works by following the established

procedures of the concerned state

government.

The district authority is empowered to

examine the eligibility of works sanction

funds and select the implementing

agencies, prioritize works, supervise

overall execution, and monitor the

scheme at the ground level.

The LokSabha Members can recommend

works in their respective constituencies.

The elected members of the RajyaSabha

can recommend works anywhere in the

state from which they are elected.

Nominated members of the LokSabha

and RajyaSabha may select works for

implementation anywhere in the

country.

Page 9: PRELIMS Academy for Civil Services PRESS · 2.13.1 Mission RakshaGyan Shakti 2.14 National Food Security Act (NFSA) 2.15 Ministry of Rural Development 2.15.1 Mission Antyodaya 2.15.2

https://t.me/joinchat/AAAAAFYrI5kpQsEAKqmo-A

Page | 8

2.6 Ministry of External Affairs

2.6.1 Know India Programme

Know India Programme launched in 2004 is an initiative of the Ministry of External Affairs for the Persons

of Indian Origin (PIO) between the age group of 18 to 30 years. The Know India Programme is a three-

week orientation programme which is aimed at promoting awareness on different facets of life in India as

well as the progress made by the country in fields such as economic and education. The KIP offers a

platform for the young PIOs to visit India share their views, expectations and experiences and forge closer

bonds with the India of present times.

2.6.2 SAMEEP

.

Students and MEA Engagement Programme (SAMEEP) was launched by Ministry of External Affairs as an

outreach mission to take Indian foreign policy and its global engagements to students across the country.

It aims to familiarise students in India about the functioning of the MEA.

All the ministry officers are asked to engage students in the schools and colleges in the jobs that the

ministry is engaged in. The programme is voluntary and gives officials the option of going back to their

alma mater or to any school or college in their hometown.

2.7 Ministry of Water Resources, River Development and Ganga

Rejuvenation

2.7.1 Namami Gange Programme

It was launched in 2015 by ministry of Water Resources, River Development and Ganga Rejuvenation

with an objective to clean and protect the Ganga river in a comprehensive manner.

It is also known as Integrated Ganga Conservation Mission Project and isa Central government Project

(100% centrally funded). It will cover 8 states & 12 rivers.

Main components include Expanding waste/sewage treatment, River Front Development, River

surface cleaning, Biodiversity, Afforestation, Public awareness, Industrial effluent monitoring and

Ganga Gram.

Interventions taken under Namami Ganga include-

Sustainable Municipal Sewage Management

Managing Sewage from Rural Areas.

Page 10: PRELIMS Academy for Civil Services PRESS · 2.13.1 Mission RakshaGyan Shakti 2.14 National Food Security Act (NFSA) 2.15 Ministry of Rural Development 2.15.1 Mission Antyodaya 2.15.2

https://t.me/joinchat/AAAAAFYrI5kpQsEAKqmo-A

Page | 9

Managing Industrial discharge and pollution abatement

Enforcing River Regulatory Zones on Ganga Banks, Restoration and Conservation of

wetlands, Efficient Irrigation methods.

Ensuring Ecological rejuvenation by conservation of aquatic life and biodiversity

Promotion of Tourism and Shipping in a rational and sustainable manner

Knowledge Management on Ganga through Ganga Knowledge Centre

2.7.2 Atal Bhujal Yojana (ABHY)

Implemented by Ministry of Water Resources, River Development and Ganga Rejuvenation it aims at

tackling ever-deepening crisis of depleting groundwater level. It is designed as a Central Sector Scheme

with a total outlay of Rs. 6,000 Crore and is proposed to be implemented with World Bank assistance.

Scheme: The scheme is to be implemented over a period of five years from 2018-19 to 2022-23,

with World Bank assistance.

The objective of scheme is to recharge ground water and create sufficient water storage for

agricultural purposes.

It also focuses on revival of surface water bodies so that groundwater level can be increased,

especially in the rural areas.

The scheme is launched in 7 water-stressed states: Gujarat, Haryana, Karnataka, Maharashtra,

Uttar Pradesh, Rajasthan and Madhya Pradesh.

These States represent about 25% of the total number of over-exploited, critical and semi-critical

blocks in terms of ground water in India. They also cover two major types of groundwater

systems found in India – alluvial and hard rock aquifers- and have varying degrees of

institutional readiness and experience in groundwater management.

2.8 Ministry of Culture

2.8.1 Seva Bhoj Yojana

The Union Ministry of Culture on June 1, 2018 introduced a new scheme called „Seva Bhoj Yojna‟ to

reimburse the central share of CGST and IGST on items for food/prasad/langar/bhandara offered free of

cost by charitable religious institutions.

Page 11: PRELIMS Academy for Civil Services PRESS · 2.13.1 Mission RakshaGyan Shakti 2.14 National Food Security Act (NFSA) 2.15 Ministry of Rural Development 2.15.1 Mission Antyodaya 2.15.2

https://t.me/joinchat/AAAAAFYrI5kpQsEAKqmo-A

Page | 10

About the Yojana:

The scheme seeks to reimburse the central government‟s share of Central Goods and Services Tax

(CGST) and Integrated Goods and Service Tax (IGST) on purchase of raw items such as ghee,

edible oil, atta, maida, rava, flour, rice pulses, sugar and jaggery, which go into preparation of

food/prasad/langar/bhandara offered free of cost by religious institutions.

The main objective of the scheme is to lessen the financial burden of such charitable religious

institutions, which provide free of cost without any discrimination to the general public and

devotees.

The institutions/organizations should have been in existence for preceding three years before

applying for assistance

2.8.2 Project Mausam

Project „Mausam‟ is the initiative of Ministry of Culture to be implemented by the Archaeological

Survey of India (ASI) as the nodal agency with research support of the Indira Gandhi National Centre

for the Arts (IGNCA) and National Museum as associate bodies

Project Mausam is to showcase a Transnational Mixed Route (including Natural and Cultural

Heritage) on the World Heritage List.

It aims to understand how the knowledge and manipulation of the monsoon winds has shaped

interactions across the Indian Ocean and led to the spread of shared knowledge systems, traditions,

technologies and ideas along maritime routes.

The project collates archaeological and historical research and document the diversity of cultural,

commercial and religious interactions among the 39 Indian Ocean countries.

2.9 The Ministry of New and Renewable Energy

2.9.1 Kusum (Kisan Urja Suraksha evam Utthaan Mahaabhiyan) Scheme

The KUSUM (Kisan Urja Suraksha evam

Utthaan Mahabhiyan) scheme was announced

in the Union Budget in 2018 with an aim to

promote use of solar power among farmers.

Under this scheme farmers would be provided

with solar water pumps.

Kisan Urja Suraksha evam Utthaan

Mahabhiyan (KUSUM)’ provides for:

1. Installation of grid-connected solar

power plants each of capacity up to 2

MW in the rural areas;

2. Installation of standalone off-grid solar

water pumps to fulfill irrigation needs of

farmers not connected to grid;

3. Solarisation of existing grid-connected

agriculture pumps to make farmers

independent of grid supply and also

enable them to sell surplus solar power

Page 12: PRELIMS Academy for Civil Services PRESS · 2.13.1 Mission RakshaGyan Shakti 2.14 National Food Security Act (NFSA) 2.15 Ministry of Rural Development 2.15.1 Mission Antyodaya 2.15.2

https://t.me/joinchat/AAAAAFYrI5kpQsEAKqmo-A

Page | 11

generated to DISCOM and get extra

income; and

4. Solarisation of tube-wells and lift

irrigation projects of Government sector.

Central Government’s plan

The government has planned to provide 27.5

lakh solar pumps (17.50 lakh standalone + 10

Lakh Grid-connected) under the scheme. It will

help farmers install total 10 GW of Solar Power

Plants of intermediate capacity of 0.5 to 2 MW

each. It also envisages 50,000 Grid-connected

tube-wells/lift irrigation and drinking water

projects.

2.10 Ministry of Commerce and Industry

2.10.1 ReUnite

The Union Minister of Commerce & Industry and Civil Aviation has launched mobile application “ReUnite”

in New Delhi to track and trace missing and abandoned children in India. The app is available for both

Android and iOS. It has been developed by Nobel Laureate Kailash Satyarthi„s NGO Bachpan Bachao

Andolan and IT company Capgemini.

2.10.2 Revenue Sharing Scheme for Plantation Crops

The Revenue Insurance Scheme for Plantation Crops (RISPC) is a Department of Commerce scheme for

protecting growers of tea, coffee, rubber, cardamom and tobacco from the twin risks of weather and price

arising from yield loss due to adverse weather parameters, pest attacks etc. and from income loss caused

by fall in international/domestic prices through crop insurance mechanism

Coverage of the scheme

1. The Insurance premium subvention under RISPC is for small growers of Rubber, Tea, Coffee

(Robusta and Arabica), Tobacco and Cardamom (small and large) having 10 ha. or less

landholding. The scheme will be applicable to mature standing crops only.

2. The Scheme is compulsory for growers registered with the respective Commodity Boards (CBs) in

the pilot districts/member growers availing themselves of benefits under other schemes of

Government through CBS and/or growers availing themselves of loans from public financial

institutions/bodies including the CBS. The Scheme is optional for other small growers. Large

growers can also participate in the scheme by paying the the actuarial premium as they are not

eligible for premium subsidy.

Page 13: PRELIMS Academy for Civil Services PRESS · 2.13.1 Mission RakshaGyan Shakti 2.14 National Food Security Act (NFSA) 2.15 Ministry of Rural Development 2.15.1 Mission Antyodaya 2.15.2

https://t.me/joinchat/AAAAAFYrI5kpQsEAKqmo-A

Page | 12

3. The scheme would be implemented initially on pilot basis in selected eight districts of seven

States. The total number of small growers expected to be covered are around 1.8 lakhs with all

area coverage of around 2.10 lakh hectares.

4. The scheme will operate on the principle of 'Area Approach' in the selected districts. Commodity

Board, in consultation with the concerned State Government shall designate an area as

Insurance Unit (IU), which can be a village/village panchayat or any other equivalent unit.

5. Scheme will cover income loss arising out of yield loss /price fluctuations or both. Yield loss can

be due to non-preventable risks viz., drought, dry spells, flood, inundation, pest and diseases,

landslides, natural fire, lightening, storm, hailstorm, cyclone etc. Price fluctuation can be due to

fall in international/domestic prices below the average price of last 5 years excluding the current

year.

6. Losses arising out of war and nuclear risks, malicious damage and other preventable risks are

excluded.

2.11 Ministry of Heavy Industries and Public Enterprises

2.11.1 FAME-India Scheme

The Faster Adoption and Manufacturing of

(Hybrid &) Electric Vehicles (FAME) India was

launched in 2015 under National Electric

Mobility Mission (NEMM). It aims at promoting

eco-friendly vehicles in the country. The scheme

is being administered by the Heavy Industries

Ministry.

Objectives

Provide fiscal and monetary incentives for

adoption and market creation of both hybrid and

electric technologies vehicles in the country and

incentivise all vehicle segments, including two-

wheelers, three wheeler auto, passenger four-

wheeler vehicle, light commercial vehicles and

buses. In order to promote manufacturing of

electric and hybrid vehicle technology and to

ensure sustainable growth of the same,

Department of Heavy Industry is implementing

FAME-India Scheme- Phase-I [Faster Adoption

and Manufacturing of (Hybrid &) Electric

Vehicles in India] from 1April 2015. The scheme,

which was initially upto 31April 2017, has been

extended upto 31March, 2019 or till Notification

of FAME-II, whichever is earlier.

The Phase-II of the Faster Adoption and

Manufacturing of (Hybrid &) Electric Vehicles

(FAME-India) Scheme proposes to give a push to

electric vehicles (EVs) in public transport and

seeks to encourage adoption of EVs by way of

market creation and demand aggregation.

Page 14: PRELIMS Academy for Civil Services PRESS · 2.13.1 Mission RakshaGyan Shakti 2.14 National Food Security Act (NFSA) 2.15 Ministry of Rural Development 2.15.1 Mission Antyodaya 2.15.2

https://t.me/joinchat/AAAAAFYrI5kpQsEAKqmo-A

Page | 13

2.12 PM Ujala Yojana

Key Points

Unnat Jyoti by Affordable Lighting for All (UJALA) is a Zero subsidy Scheme launched by the

Government of India in an effort to popularize the message of energy efficiency in the country.

The scheme is being implemented by a Public Sector Undertaking of the Government of India,

Energy Efficiency Services Limited (EESL) under the Union Ministry of Power and the Electricity

Distribution Company.

It is a LED based Domestic Efficient Lighting Programme (DELP) targeting promotion of reduced

energy consumption, energy savings and efficient lighting. Under the scheme, Electricity

Distribution Company would distribute LED bulbs at subsidized rates to every grid-connected

customer with a metered connection

2.13 Ministry of Defence

2.13.1 Mission Raksha Gyan Shakti

Union Defence Minister launched „Mission Raksha Gyan Shakti‟ in New Delhi to boost Intellectual

Property Right (IPR) culture in indigenous defence industry. It was instituted by Department of Defence

Production as part of the ongoing initiative to enhance self-reliance in defence. It aimed to provide a boost

to IPR culture in indigenous defence industry. The main objective of this programme was to inculcate IP

culture in Indian defence manufacturing ecosystem. Directorate General of Quality Assurance (DGQA)

was entrusted with responsibility of coordinating and implementing the programme.

2.14 National Food Security Act (NFSA)

About National Food Security Act (NFSA), 2013

The NFSA aims to provide subsidized food grains to approximately two thirds of the population (75% in

rural areas and 50% in urban areas).Grains like wheat, rice and coarse grain will be distributed at the

subsidized price of Rs. 3, Rs. 2 and Rs. 1. Pregnant women and lactating mothers and children are

entitled to get meals under the prescribed nutrition by MDM and ICDS. NFSA 2013 will provide high

nutrition food to the children from age group of 6 months to 14 years. Pregnant women and lactating

mothers will be entitled to get maternity benefit of not less than Rs. 6,000

Facts:

It is now being implemented across entire country

Page 15: PRELIMS Academy for Civil Services PRESS · 2.13.1 Mission RakshaGyan Shakti 2.14 National Food Security Act (NFSA) 2.15 Ministry of Rural Development 2.15.1 Mission Antyodaya 2.15.2

https://t.me/joinchat/AAAAAFYrI5kpQsEAKqmo-A

Page | 14

81.34 crore persons will get subsidised wheat at Rs. 2 per kg and rice at Rs. 3 per kg.

At the current coverage, the monthly allocation of food grains to States and UTs under the Act is

about 45.5 lakh tonnes.

The subsidy implication of about Rs. 11,726 crore per month and it is overall about Rs. 1,40,700

crore per year.

Now Union Government will focus on further reforms in the public distribution system (PDS)

including end to end computerization in order to check leakages and diversion of food grains.

2.15 Ministry of Rural Development

2.15.1 Mission Antyodaya

Key Points:

Under the mission, Department of Rural Development in partnership with State Governments is

involved in ranking 50,000 Gram Panchayats.

The ranking is based on parameters of physical infrastructure, human development and

economic activities.

It facilitates identification of gaps in a quest for poverty free gram panchayats and drive economic

activities.

Public institutions like Krishi Vigyan Kendras, MSME Clusters will be involved for enhancing

productive employment and economic activities.

2.15.2 Shyama Prasad Mukherji Rurban Mission

Mission’s Objective:

The objective of the National Rurban Mission (NRuM) is to stimulate local economic development, enhance

basic services, and create well planned Rurban clusters.

There is a funding support of up to 30% of the estimated investment for each Rurban cluster, given as

Critical Gap Funding (CGF), while 70% of the funds is mobilized by the States through convergence with

synergic State and Central programmes and private investment and institutional funding. The CGF is now

shared between the Centre and the State in a ratio of 60:40 for Plain area States and 90:10 for Himalayan

and NE States.

Various Provisions:

Provision of basic amenities

Provision of 24/7 Water Supply to all households

Solid and Liquid Waste Management facilities at the household and cluster level

Page 16: PRELIMS Academy for Civil Services PRESS · 2.13.1 Mission RakshaGyan Shakti 2.14 National Food Security Act (NFSA) 2.15 Ministry of Rural Development 2.15.1 Mission Antyodaya 2.15.2

https://t.me/joinchat/AAAAAFYrI5kpQsEAKqmo-A

Page | 15

Provision of Inter and Intra village roads –

Adequate Street Lights and Public Transport facilities using green technologies

Provision of Economic Amenities –

Various thematic areas in the sectors of Agri Services and Processing, Tourism, and Skill

development to promote Small and Medium Scale Enterprises

2.15.3 Gram Swaraj Abhiyan

The Cabinet Committee on Economic Affairs

gave its approval for restructured Centrally

Sponsored Scheme of Rashtriya Gram Swaraj

Abhiyan (RGSA) with the objective to make rural

local bodies self-sustainable, financially stable

and more efficient.

It addresses critical gaps that hinder the success

of panchayats by

Enhancing their capacities and

effectiveness

Promoting devolution of powers and

responsibilities

Centre-State Responsibility:

The sharing ratio for the state

components will be 60:40 barring the

Northeast and hilly states where it will

be 90:10.

For UTs, the central share will be 100

per cent.

The Central component includes

national level activities such as „National

Plan of Technical Assistance‟, „Mission

Mode project on e-Panchayat‟,

„Incentivisation of Panchayats‟; while the

State component includes „Capacity

Building of Panchayati Raj Institutions

(PRIs)‟.

The State Governments will formulate

the Annual Action Plans for seeking

assistance from the Central

Government.

Nature of the Scheme

Priority will be given to subjects of

national importance that affects the

excluded groups the most, e.g. poverty,

primary health services, nutrition,

immunization, sanitation, education,

water conservation, digital transactions

etc.

The scheme is designed keeping in view

programmatic convergence with Mission

Antyodaya Gram Panchayats and 115

Aspirational districts as identified by

NITI Aayog. As Panchayats have

representation of Schedule Castes,

Schedule Tribes and women, and are

institutions closest to the grassroots,

strengthening Panchayats will promote

equity and inclusiveness, along with

Social Justice and economic

development of the community.

Page 17: PRELIMS Academy for Civil Services PRESS · 2.13.1 Mission RakshaGyan Shakti 2.14 National Food Security Act (NFSA) 2.15 Ministry of Rural Development 2.15.1 Mission Antyodaya 2.15.2

https://t.me/joinchat/AAAAAFYrI5kpQsEAKqmo-A

Page | 16

2.15.4 Deendayal Upadhyaya Grameen Kaushalya Yojana (DDU-GKY)

The Ministry of Rural Development (MoRD) announced the Deen Dayal Upadhyaya Grameen Kaushalya

Yojana (DDU-GKY) Antyodaya Diwas, on 25th September 2014. DDU-GKY is a part of the National Rural

Livelihood Mission (NRLM), tasked with the dual objectives of adding diversity to the incomes of rural poor

families and cater to the career aspirations of rural youth.

The Vision of DDU-GKY is to "Transform rural poor youth into an economically independent and globally

relevant workforce". It aims to target youth, in the age group of 15–35 years.

2.15.5 Pradhan Mantri Awas Yojana (Rural)

In pursuance to the goal - Housing for all by 2022, the rural housing scheme Indira Awas Yojana has

been revamped to Pradhan Mantri Awaas Yojana (Gramin) w.e.f. 1April 2016. Under the scheme, financial

assistance is provided for construction of pucca house to all houseless and households living in

dilapidated houses in rural areas.

Target

Under the scheme, it is proposed to build four crore pucca houses in total, by the year 2022. One

crore households would be provided assistance for construction of pucca house during the first

phase, from 2016-17 to 2018-19.

Identification of beneficiaries

To ensure that assistance is targeted at those who are genuinely deprived PMAY-G selects beneficiary

using housing deprivation parameters in the Socio-Economic and Caste Census (SECC), 2011, which is to

be verified by the Gram Sabhas.

Funding

The cost of unit assistance is to be shared between Central and State Governments in the ratio 60:40 in

plain areas and 90:10 for North Eastern and hilly states.

Financial Assistance - The unit assistance given to beneficiaries under the programme is Rs 1,20,000 in

plain areas and to Rs 1,30,000 in hilly states/difficult areas /Integrated Action Plan (IAP) for Selected

Tribal and Backward Districts

Page 18: PRELIMS Academy for Civil Services PRESS · 2.13.1 Mission RakshaGyan Shakti 2.14 National Food Security Act (NFSA) 2.15 Ministry of Rural Development 2.15.1 Mission Antyodaya 2.15.2

https://t.me/joinchat/AAAAAFYrI5kpQsEAKqmo-A

Page | 17

2.15.6 Deendayal Antyodaya Yojana – National Rural Livelihood Mission

(DAY-NRLM)

The initial scheme Swarnajayanti Gram Swarozgar Yojana (SGSY) was launched in 1999. It was

renamed as National Rural Livelihood Mission (NRLM) in 2011. In November 2015, the program

was renamed Deendayal Antyodaya Yojana (DAY-NRLM).

It is aided in part through investment support by the World Bank, the Mission aims at creating

efficient and effective institutional platforms of the rural poor, enabling them to increase

household income through sustainable livelihood enhancements and improved access to financial

services.

NRLM set out with an agenda to cover 7 Crore rural poor households, across 600 districts, 6000

blocks, 2.5 lakh Gram Panchayats and 6 lakh villages in the country through self-managed Self

Help Groups (SHGs) and federated institutions and support them for livelihoods collectives in a

period of 8-10 years.

A loan of Rs.2 lakh (US$2,800) each will be provided to individuals and Rs.10 lakh (US$14,000)

each to groups for setting up micro-enterprises

2.16 Ministry of Social Justice and Empowerment

2.16.1 Rashtriya Vayoshri Yojana (RVY)

Key Points:

The Ministry of Social Justice and Empowerment has introduced a scheme for providing physical

aids and Assisted Living Devices for Senior Citizens belonging to BPL category named “Rashtriya

Vayoshri Yojana (RVY)” on 1st April, 2017 with the objective of providing Senior Citizens, belonging

to BPL category and suffering from age related disabilities/ infirmities, with such physical aids

and assisted living devices which can restore near normalcy in their bodily functions.

It is the first-of-its-kind Central Sector Scheme (CCS) in India, to be fully funded by the Union

Government.

Identification of beneficiaries: It will be done by the State Governments/UTs through a Committee

chaired by the Deputy Commissioner/District Collector. 30% of the beneficiaries in each district

shall be women. The Scheme is being implemented through the “Artificial Limbs Manufacturing

Corporation (ALIMCO)”, a Public Sector Undertaking under the Ministry of SJ&E, as the sole

Implementing Agency.

Page 19: PRELIMS Academy for Civil Services PRESS · 2.13.1 Mission RakshaGyan Shakti 2.14 National Food Security Act (NFSA) 2.15 Ministry of Rural Development 2.15.1 Mission Antyodaya 2.15.2

https://t.me/joinchat/AAAAAFYrI5kpQsEAKqmo-A

Page | 18

2.17 Ministry of Micro, Small and Medium Enterprises

2.17.1 Udyam Sakhi Portal

Key Points:

On the occasion of International Women‟s Day today, March 8, the Ministry of Micro, Small and

Medium Enterprises (MSME) launched a portal for women entrepreneurs of India.

The portal is a network for nurturing entrepreneurship and creating business models for low cost

products and services in order to empower women and make them self-reliant and self-sufficient.

The portal provides assistance through its platform for entrepreneurship learning tools,

incubation facility, training programs for fund raising, providing mentors, one-on-one investor

meet, provide market survey facility and technical assistance.

2.17.2 Prime Minister's Employment Generation Programme (PMEGP)

PMEGP is implemented by Ministry of Micro, Small and Medium Enterprises with an Objective to promote

entrepreneurship and generate sustainable employment. Khadi and Village Industries Commission (KVIC)

is nodal implementation agency at national level. At State and district level, State offices of KVIC, Khadi

and Village Industries Boards (KVIBs) and District Industry Centres (DIC) are the implementing agencies.

Scheme is being implemented from 2008-2009.

Scheme:

PMEGP is the flagship programme of the government offering credit linked subsidy to

establish new enterprises for generating continuous and sustainable employment

opportunities in Rural and Urban areas of the country.

There is no income ceiling for setting up the project. It is aimed at generating self-

employment opportunities through establishment of micro-enterprises in non-farm sector by

helping traditional artisans and unemployed youth in rural as well as urban areas.

2.18 Ministry of Petroleum and Natural Gas

2.18.1 Ujjwala Programme

It was launched by Ministry of Petroleum and Natural Gas with an objective to reduce health hazards of

indoor pollution by providing free LPG connections to Women from BPL Households

Page 20: PRELIMS Academy for Civil Services PRESS · 2.13.1 Mission RakshaGyan Shakti 2.14 National Food Security Act (NFSA) 2.15 Ministry of Rural Development 2.15.1 Mission Antyodaya 2.15.2

https://t.me/joinchat/AAAAAFYrI5kpQsEAKqmo-A

Page | 19

Tagline: Swachh Indhan, Behtar Jeevan

Scheme:

8 Crore (Earlier target was 5 Crore) LPG (liquefied petroleum gas) connections to poor households will

be provided. The scheme provides free LPG connection with financial assistance of Rs. 1600/- per

connection to an adult woman member of BPL family identified through Socio-Economic Caste

Census (SECC) data. Eligible households will be identified in consultation with state governments

and Union territories. The scheme will be implemented by 2020. (Older target was to be achieved by

2019).

The households will be selected using the socio-economic and caste census data. Consumers will

have the option to purchase gas stove and refills on EMI.

It seeks to empower women and protect their health by shifting them from traditional cooking based

on unclean cooking fuels or fossil fuels to clean cooking gas.

2.18.2 Sustainable Alternative Towards Affordable Transportation (SATAT)

Ministry of Petroleum & Natural Gas has launched Sustainable Alternative Towards Affordable

Transportation (SATAT) aimed at providing sustainable alternative towards affordable transportation. The

initiative was launched with PSU Oil Marketing Companies (OMCs i.e. IOC, BPCL and HPCL).

The SATAT initiative has potential to boost availability of more affordable transport fuels, better

use of agricultural residue, cattle dung and municipal solid waste, as well as provide additional

revenue source to farmers. It will boost availability of more affordable transport fuels and enable

better use of agricultural residue, cattle dung and municipal solid waste. It will pave way for

efficient municipal solid waste management and help in tackling problem of polluted urban air

due to farm stubble-burning and carbon emissions.

2.18.3 LPG Panchayat

Key Points:

Pradhan Mantri LPG Panchayat scheme is a nation-wide scheme launched by the Ministry of

Petroleum and Natural gas designed to distribute LPG connections to the rural households where

conventional fuel is used for domestic purposes and to give a boost to the existing Pradhan

Mantri Ujjwala Yojana (PMUY).

LPG Panchayat is a backup scheme to the existing PMUY.

The scheme aims to create awareness about the benefits of replacing the conventional fuels like

cow-dung, wood, charcoal etc with LPG. It was introduced to prevent lakhs of deaths due to

health complications arising out of use of conventional fuels. Under the scheme Asha workers,

officials of Public Sector Undertakings, NGOs and social workers will be involved in connecting

the beneficiaries to the Ujjwala Yojana. They would also take steps to resolve issues and

traditional beliefs among the people.

Page 21: PRELIMS Academy for Civil Services PRESS · 2.13.1 Mission RakshaGyan Shakti 2.14 National Food Security Act (NFSA) 2.15 Ministry of Rural Development 2.15.1 Mission Antyodaya 2.15.2

https://t.me/joinchat/AAAAAFYrI5kpQsEAKqmo-A

Page | 20

2.19 Ministry of Finance

2.19.1 Pradhan Mantri Vaya Vandana Yojana (PMVVY)

Key Points

PMVVY is a Pension Scheme under the Ministry of Finance, exclusively for the senior citizens

aged 60 years and above.

Life Insurance Corporation of India is the implementing agency. It enables assured

pension/return linked to the subscription amount based on government guarantee to LIC.

It provides an assured return of 8% p.a. payable monthly for 10 years and is exempted from

Service Tax/ GST.

The ceiling of maximum pension is for a family as a whole, the family will comprise of pensioner,

his/her spouse and dependents. The shortfall owing to the difference between the interest

guaranteed and the actual interest earned shall be subsidized by the Government of India and

reimbursed to the Corporation.

2.19.2 Atal Pension Yojana

It is implemented by Ministry of Finance

with an objective to address the longevity

risks among the workers in unorganised

sector and to encourage the workers in

unorganised sector to voluntarily save for

their retirement.

It is administered by the Pension Fund

Regulatory and Development Authority. The

minimum age of joining APY is 18 years and

maximum age is 65 years. Minimum period

of contribution by the subscriber under APY

would be 20 years or more It was launched

in 2015.

Atal Pension Yojana (APY) is open to all bank

account holders who are not members of

any statutory social security scheme.

Subscribers would receive the fixed pension

of Rs. 1K/2K/3K/4K/5K per month, at the

age of 60 years, depending on their

contributions.

The Central Government would also co-

contribute 50% of the subscriber‟s

contribution or Rs. 1000 per annum,

whichever is lower, to each eligible

subscriber account, for a period of 5 years.

There is no exit to the scheme before the age

of 60. In case of death of subscriber, the

spouse of the subscriber shall be entitled for

the same amount of pension till his or her

death.

New Changes:

All accounts opened after August 2018

will have accident insurance limit of Rs

2 lakh, double than earlier Rs 1 lakh

limit. The overdraft facility of the scheme

was also increased from Rs 5,000 to Rs

10,000. Earlier, people of age 18 to 60

years were entitled to enroll in this

scheme. But now it has been relaxed

further to 65 years, taking into

consideration rise in average age-

expectancy.

Page 22: PRELIMS Academy for Civil Services PRESS · 2.13.1 Mission RakshaGyan Shakti 2.14 National Food Security Act (NFSA) 2.15 Ministry of Rural Development 2.15.1 Mission Antyodaya 2.15.2

https://t.me/joinchat/AAAAAFYrI5kpQsEAKqmo-A

Page | 21

2.19.3 PradhanMantri Jan DhanYojana

Central Govt. has decided to make Pradhan Mantri Jan Dhan Yojana an Open Ended Scheme. Under this

PMJDY Scheme, govt. has now added more number of incentives in order to encourage poor people to

open zero balance bank accounts. This scheme was launched in August 2014 for 4 years. 1st phase of

PMJDY scheme was completely focused on opening basic bank accounts. It also provides RuPay debit

cards with inbuilt accident insurance cover of Rs. 100,000.

Other Features:

Accidental Insurance Now Rs. 2 Lakh – All the people who opens new Jan Dhan Bank Accounts

after 28 August 2018 will now get Free Accident Insurance Cover of double amount of Rs. 2 lakh.

Over-draft Limit Now Rs. 10,000 – Now there would be no conditions attached for overdraft of

upto Rs. 2,000. Now the maximum limit for over-draft is set at Rs. 10,000 (previously Rs. 5,000).

In PMJDY, this facility is available after 6 months of opening bank accounts.

Upper Age Limit now 65 Years – Also the upper age limit to avail the Jan Dhan Yojana facility is

now raised from 60 years to 65 years.

2.19.5 Pradhan Mantri Mudra Yojana

It was launched in 2015 by Ministry of Finance with an objective to fund the unfunded. It will finance to

“Last Mile Financiers” of small/micro businesses. The lending priority will be given to SC/ST enterprises

MUDRA Bank is set up as a statutory body. It regulates and refinances all MFI who lend to MSME

engaged in small manufacturing, trade or services. It will partner all state/regional level coordinators to

provide easy finance to even the remote investors.

MUDRA Bank has three loan instruments:

o Shishu: covers loans uptoRs. 50,000/-

o Kishor: covers loans above Rs. 50,000/- and uptoRs. 5 lakh

o Tarun: covers loans above Rs. 5 lakh and uptoRs. 10 lakh

It provides a loan at low rates to small entrepreneurs

The bank has been allotted a Refinance Fund of Rs. 20,000 Crores from the shortfalls of Priority

Sector Lending.

2.20 Ministry of Minority Affairs

2.20.1 PradhanMantri Jan VikasKaryakram (PMJVK)

The Programme aims to address development deficits in the identified minority concentration

areas. The identification of minority concentration areas has been done on the basis of presence

of substantial population of notified Minority Communities based on Census, 2011.

Page 23: PRELIMS Academy for Civil Services PRESS · 2.13.1 Mission RakshaGyan Shakti 2.14 National Food Security Act (NFSA) 2.15 Ministry of Rural Development 2.15.1 Mission Antyodaya 2.15.2

https://t.me/joinchat/AAAAAFYrI5kpQsEAKqmo-A

Page | 22

The restructured programme would provide better socio economic infrastructure facilities to the

minority communities particularly in the field of education, health & skill development as

compared to the present situation, which would further lead to lessening of the gap between the

national average and the minority communities with regard to backwardness parameters.

States/Districts covered:

The PMJVK will be implemented in Minority Concentration District Headquarters, Minority

Concentration Block (MCBs), Minority Concentration Towns(MCTs) falling in 308 districts of 32

States/UTs.

Moreover, the scheme will also be implemented in Backward Clusters of Minority Concentration

Villages (CoMCV). These CoMCV will be identified on the proposal of the States/UTs as per

criteria of PMJVK.

2.20.2 Jiyo Parsi Scheme

Jiyo Parsi scheme is a Central Sector Scheme for containing population decline of Parsis in India. Its

main objective is to reverse the declining trend of Parsi population by adopting a scientific protocol and

structured interventions. It aims to stabilize and increase the population of Parsis in India.

It has two components: Medical Assistance and Advocacy (Counseling). The first phase of the scheme was

launched initiated in 2013.

2.21 Ministry of Women and Child Development

2.21.1 POSHAN Abhiyan

POSHAN Abhiyan was launched by Prime

Minister Narendra Modi in Jhunjhunu,

Rajasthan in March 2018.

It is implemented by Ministry of Women

and Child Development and aims at

ensuring holistic development and

adequate nutrition for pregnant women,

mothers and children.

The Union Government has signed $200

million loan agreement with World Bank

for National Nutrition Mission (POSHAN

Abhiyan) for 315 districts across all

states and union territories. The loan

proceeds will be used for reducing

stunting in children 0-6 years of age

from 38.4% to 25% by 2022 under

POSHAN Abhiyan.

Scheme:

It targets to reduce level of under-

nutrition and other related problems by

ensuring convergence of various

nutrition related schemes.

It also targets stunting, under-nutrition,

anaemia (among young children, women

and adolescent girls) and low birth rate.

It will monitor and review

implementation of all such schemes and

Page 24: PRELIMS Academy for Civil Services PRESS · 2.13.1 Mission RakshaGyan Shakti 2.14 National Food Security Act (NFSA) 2.15 Ministry of Rural Development 2.15.1 Mission Antyodaya 2.15.2

https://t.me/joinchat/AAAAAFYrI5kpQsEAKqmo-A

Page | 23

utilize existing structural arrangements

of line ministries wherever available.

Its large component involves gradual

scaling-up of interventions supported by

ongoing World Bank assisted Integrated

Child Development Services (ICDS)

Systems Strengthening and Nutrition

Improvement Project (ISSNIP) to all

districts in the country by 2022.

2.21.2 Mahila Shakti Kendra Scheme

Government of India has approved a new scheme namely Mahila Shakti Kendra for 2017-18 upto 2019-

20 with the objective to empower the rural women to realize their full potential by means of community

participation. The scheme is envisaged to work at various levels, and at the National level (domain based

knowledge support) and State level (State Resource Centre for Women) technical support to the respective

governments on issues related to women is provided.

PMMSK scheme is envisioned as one-stop convergence support service for empowering rural women with

opportunities for skill development, digital literacy, health and nutrition and employment. It aims to

improve declining child sex ratio (CSR), ensure survival and protection of the girl child, ensuring her

education and empowering her to fulfill her potential.

Through this scheme, government plans to reach 115 most backward districts in the country with 920

Mahila Shakti Kendra. The government plans to reach the 115 most backward districts in the country

with 920 Mahila Shakti Kendra

2.21.3 SHe box

It is an online complaint management system for registering complaints related to sexual harassment at

workplace. It was launched by the Ministry of Women and Child Development. The complaint

management system has been developed to ensure the effective implementation of Sexual Harassment of

Women at Workplace (Prevention, Prohibition and Redressal) Act, 2013.

How it works?

Once a complaint is submitted to the portal, it will be directly sent to the Internal Complaints Committee

(ICC) of the concerned Ministry/Department/PSU/Autonomous Body etc. having jurisdiction to inquire

into the complaint. Through this portal, WCD as well as complainant can monitor the progress of inquiry

conducted by the ICC.

Page 25: PRELIMS Academy for Civil Services PRESS · 2.13.1 Mission RakshaGyan Shakti 2.14 National Food Security Act (NFSA) 2.15 Ministry of Rural Development 2.15.1 Mission Antyodaya 2.15.2

https://t.me/joinchat/AAAAAFYrI5kpQsEAKqmo-A

Page | 24

2.21.4 Swadhar Greh Scheme

The Ministry of Women and Child Development is implementing the Swadhar Greh Scheme which targets

the women victims of difficult circumstances who are in need of institutional support for rehabilitation so

that they could lead their life with dignity. The Scheme envisages providing shelter, food, clothing and

health as well as economic and social security for these women. Swadhar Greh is a DBT compliant

scheme.

As per guidelines of the Swadhar Greh Scheme, to seek financial assistance the agency should

meet following requirements:

The agency should be either recognized by State/UT under existing law or should be well

known with the experience or working in the field for at least 3 years and its work should be

reported satisfactory by the State Govt./UT Administration concerned.

It should ordinarily have been engaged in the field of women‟s welfare/social welfare for a

minimum period of two years.

Its financial position should be sound.

It should have facilities, resources, experience and personnel to undertake the management

of such projects.

It should run Swadhar Greh on a no-profit basis and

It should have facilities like computers, internet connection etc at Swadhar Greh.

2.21.5 Beti Bachao Beti Padhao(BBBP)

The campaign was launched by Prime Minister in January 2015 at Panipat, Haryana as comprehensive

programme to address declining Child Sex Ratio (CSR) and related issues of empowerment of women over

life-cycle continuum. The specific objectives of scheme are preventing gender biased sex selective

elimination, ensuring survival and protection of the girl child and ensuring education and participation of

the girl child. Its focus is on awareness and advocacy campaign, multi-sectoral action enabling girls‟ girls‟

education and effective enforcement of Pre-Conception & Pre Natal Diagnostic Techniques (PC&PNDT) Act.

The scheme is being implemented as a tri-ministerial, convergent effort of Union Ministries of

Women and Child Development (WCD), Health & Family Welfare (MoHFW) and Human Resource

Development (HRD). The Union ‎Ministry of Women and Child Development (WCD) is nodal ministry

for programme at central level.

The Union Government has expanded Beti Bachao Beti Padhao (BBBP) programme from 161 districts to

all 640 districts of the country. The initial focus of BBBP was limited to districts which were either below

national average or were worse in their own states in terms of absolute values of Child Sex Ratio.

Page 26: PRELIMS Academy for Civil Services PRESS · 2.13.1 Mission RakshaGyan Shakti 2.14 National Food Security Act (NFSA) 2.15 Ministry of Rural Development 2.15.1 Mission Antyodaya 2.15.2

https://t.me/joinchat/AAAAAFYrI5kpQsEAKqmo-A

Page | 25

2.21.6 Pradhan Mantri Matru Vandana Yojana (PMMVY)

Key Points:

Pradhan Mantri MatruVandana Yojana (PMMVY) is a maternity benefit rechristened from

erstwhile Indira Gandhi Matritva Sahyog Yojana (IGMSY). The IGMSY was launched in 2010.

The scheme is a Centrally Sponsored Scheme under which cost sharing ratio between the Centre

and the States & UTs with Legislature is 60:40 while for North-Eastern States & three Himalayan

States; it is 90:10. It is 100% Central assistance for Union Territories without Legislature.

The scheme is a conditional cash transfer scheme for pregnant and lactating women of 19 years

of age or above for first live birth. It provides partial wage compensation to women for wage-loss

during childbirth and childcare and to provide conditions for safe delivery and good nutrition and

feeding practices.

The maternity benefits under Pradhan Mantri Matru Vandana Yojana (PMMVY) are available to all

Pregnant Women & Lactating Mothers (PW&LM) except those in regular employment with the

Central Government or State Government or Public Sector Undertaking or those who are in

receipt of similar benefits under any law for the time being in force.

2.22 Ministry of IT and Electronics

2.22.1 Project Stree Swabhimaan

It was launched by Ministry of IT and Electronics aiming to create sustainable model for providing

adolescent girls and women an access to affordable sanitary products by leveraging Common Service

Centres (CSCs).

Scheme:

Under this initiative, Common Service Centres (CSC) will provide access to affordable, reliable and

modern (eco-friendly) sanitary napkins (menstrual pads) to adolescent girls and women in rural

areas.

Its purpose is to improve awareness on menstrual health and hygiene of women, thus help to

protect dignity and rights of women.

Under this initiative, semi-automatic and manual sanitary napkin manufacturing units will be set

up at CSC for producing affordable and eco-friendly sanitary napkins.

These micro manufacturing units will be operated by women entrepreneurs and generate

employment for 8-10 women.

Page 27: PRELIMS Academy for Civil Services PRESS · 2.13.1 Mission RakshaGyan Shakti 2.14 National Food Security Act (NFSA) 2.15 Ministry of Rural Development 2.15.1 Mission Antyodaya 2.15.2

https://t.me/joinchat/AAAAAFYrI5kpQsEAKqmo-A

Page | 26

2.23 Ministry of Human Resource Development

2.23.1 Rashtriya Avishkar Abhiyan (RAA)

In pursuance of the focus on connecting school based knowledge to life outside the school and making

learning of Science Mathematics a joyful and meaningful activity, to bring focus on innovation and use of

technology, the Ministry of Human Resource Development has set up the Rashtriya Avishkar Abhiyan

(RAA)-a convergent framework that aims at nurturing a spirit of inquiry and creativity, love for Science

and Mathematics and effective use of technology amongst children and encourage those who show an

inclination and talent for these subjects to be encouraged and supported to heights of academic

excellence and research. Rashtriya Avishkar Abhiyan will target students in the age group of 6 -18 years

and inturnthe execution of RAA will span across MHRD‟s schematic interventions of Sarva Shiksha

Abhiyan, Rashtriya Madhyamik Shiksha Abhiyan In the Department of School Education & Literacy and

programmes and schemes of Department of Higher Education to encourage Science, Mathematics &

Technology.

Page 28: PRELIMS Academy for Civil Services PRESS · 2.13.1 Mission RakshaGyan Shakti 2.14 National Food Security Act (NFSA) 2.15 Ministry of Rural Development 2.15.1 Mission Antyodaya 2.15.2

https://t.me/joinchat/AAAAAFYrI5kpQsEAKqmo-A

Page | 27

2.23.2 Unnat Bharat Abhiyan 2.0

Vision

Unnat Bharat Abhiyan is inspired by the vision

of transformational change in rural development

processes by leveraging knowledge institutions

to help build the architecture of an Inclusive

India.

Mission

The Mission of Unnat Bharat Abhiyan is to

enable higher educational institutions to work

with the people of rural India in identifying

development challenges and evolving appropriate

solutions for accelerating sustainable growth. It

also aims to create a virtuous cycle between

society and an inclusive academic system by

providing knowledge and practices for emerging

professions and to upgrade the capabilities of

both the public and the private sectors in

responding to the development needs of rural

India.

Goals

To build an understanding of the

development agenda within institutes of

Higher Education and an institutional

capacity and training relevant to

national needs, especially those of rural

India

To provide rural India and regional

agencies with access to the professional

resources of the institutes of higher

education, especially those that have

acquired academic excellence in the field

of science, engineering and technology,

and management.

Major Areas of Intervention

In order to move towards the holistic

development of the villages, there are two major

domains, i.e. human development and material

(economic) development, which need to be

developed in an integrated way.

The major components of these two domains are

given below

Human development

Health

Education and culture

Values and perception development

Skills and entrepreneurship

Material (economic) development

Organic agriculture and cow-based

economy

Water management and conservation

Renewable energy sources

Artisans and rural industries

Development and harnessing of local

natural resources

Basic amenities

E-support(IT-enabling)

Page 29: PRELIMS Academy for Civil Services PRESS · 2.13.1 Mission RakshaGyan Shakti 2.14 National Food Security Act (NFSA) 2.15 Ministry of Rural Development 2.15.1 Mission Antyodaya 2.15.2

https://t.me/joinchat/AAAAAFYrI5kpQsEAKqmo-A

Page | 28

2.23.3 Sustainable Action for Transforming Human Capital in Education

(SATH-E) Programme

Objective

The main objective of the SATH-E project is to make the entire governmental school education system

responsive, aspirational and transformational for every child. It aims to create role model states for

education and mainstream islands of excellence across the country to facilitate qualitative and

quantitative transformation of learning outcomes.

Key Highlights

The comprehensive roadmaps are for the coming two years, from 2018 to 2020.

The prints lay out detailed interventions that will be taken up by the three participating states-

Jharkhand, Madhya Pradesh and Odisha – which aim to become „role model states‟ in school

education.

These roadmaps present the first-of-its-kind, customised, action-oriented programmes, outlining

interventions at the individual, district and state level.

The plans were jointly prepared by NITI Aayog, the three States and the knowledge partners of the

SATH Initiatives, the Boston Consulting Group (BCG) and Piramal Foundation for Education

Leadership (PFEL).

Funding of the Project

The initiative will be funded through a cost-sharing mechanism between NITI Aayog and the participating

states.

The Boston Consulting Group (BCG) and Piramal Foundation for Education Leadership (PFEL) were

chosen as knowledge partners for the project facilitating review, data collection and implementation

2.23.4 Integrated Scheme for School Education: Samagra Shiksha Scheme

The Government of India has launched Samagra

Shiksha - An Integrated Scheme for School

Education, w.e.f. 2018-19, which is an

overarching programme for the school education

sector extending from pre-school to class XII and

aims to ensure inclusive and equitable quality

education at all levels of school education. It

envisages the „school‟ as a continuum from pre-

school, primary, upper primary, secondary to

senior secondary levels and subsumes the three

erstwhile centrally sponsored schemes i.e. Sarva

Shiksha Abhiyan (SSA), Rashtriya Madhyamik

Shiksha Abhiyan (RMSA) and Teacher Education

(TE).

Bridging gender and social category gaps at all

levels of school education is one of the major

objectives of the scheme. The scheme reaches

out to girls and children belonging to Scheduled

Page 30: PRELIMS Academy for Civil Services PRESS · 2.13.1 Mission RakshaGyan Shakti 2.14 National Food Security Act (NFSA) 2.15 Ministry of Rural Development 2.15.1 Mission Antyodaya 2.15.2

https://t.me/joinchat/AAAAAFYrI5kpQsEAKqmo-A

Page | 29

Caste (SC), Scheduled Tribe (ST), minority

communities and transgender. The scheme also

gives attention to urban deprived children,

children affected by periodic migration and

children living in remote and scattered

habitations. Under the scheme, provision has

been made for giving preference to Special Focus

Districts (SFDs), Educationally Backward Blocks

(EEBs), LWE affected districts, and aspirational

districts while planning interventions like setting

up of primary schools, upper primary schools,

construction of additional classrooms, toilets,

Kasturba Gandhi Balika Vidyalayas (KGBVs).

2.24 Ministry of Tribal Affairs

2.24.1 Van Dhan Scheme

Implemented by Ministry of Tribal Affairs it aims

at economic development of tribals involved in

collection of Minor Food Produces (MFPs) by

helping them in optimum utilization of natural

resources and provide them sustainable

livelihood.

Scheme:

Under it, 10 Self Help Groups (SHGs) of

30 Tribal gatherers will be constituted.

The SHGs will then be trained and

provided with working capital to add

value to products they collect from

forest.

They will be able to market their

products, by working under leadership

of collector, not only within states but

also outside states.

TRIFED will provide all required training

and technical support to SHGs.

They will be trained on sustainable

harvesting, collection, primary

processing and value addition.

They will be formed into clusters to

aggregate their stock in tradable

quantity and link them with facility of

primary processing in Van Dhan Vikas

Kendra.

Van Dhan Vikas Kendra will be

established under scheme which will

provide skill upgradation and capacity

building training and setting up of

primary processing and value addition

facility.

It will be used for training of

beneficiaries by providing them

equipment and tools for primary level

processing and infrastructure and

building for housing.

The SHGs will supply their stock after

primary processing to State

Implementing Agencies or can directly

tie up with corporate secondary

processor.

Big corporates will create secondary

level value addition facility at district

level and tertiary level value addition

facility at state level under the PPP

model.

The PPP model will be based on utilizing

Private entrepreneur skills in

undertaking processing as well as

marketing of the produce.

Page 31: PRELIMS Academy for Civil Services PRESS · 2.13.1 Mission RakshaGyan Shakti 2.14 National Food Security Act (NFSA) 2.15 Ministry of Rural Development 2.15.1 Mission Antyodaya 2.15.2

https://t.me/joinchat/AAAAAFYrI5kpQsEAKqmo-A

Page | 30

The central and state governments will

provide necessary support by creating

infrastructure and providing enabling

environment for undertaking value

addition of systematic scientific lines

2.25 PM-AASHA

The Centre recently launched the Pradhan Mantri Annadata Aay Sanrakshan Abhiyan (PM-AASHA) to

ensure better prices for farm produce.

What is PM-AASHA?

The three schemes that are part of AASHA are Price Support Scheme (PSS), Price Deficiency Payment

Scheme (PDPS) and Pilot of Private Procurement and Stockist Scheme (PPPS)

These three components will complement the existing schemes of the Department of Food and

Public Distribution.

They relate to paddy, wheat and other cereals and coarse grains where procurement is at MSP

now.

PSS - Under the PSS, physical procurement of pulses, oilseeds and copra will be done by Central Nodal

Agencies. Besides, NAFED and Food Corporation of India will also take up procurement of crops under

PSS. The expenditure and losses due to procurement will be borne by the Centre.

PDPS - Under the PDPS, the Centre proposes to cover all oilseeds. The difference between the MSP and

actual selling/modal price will be directly paid into the farmer's bank account. Farmers who sell their

crops in recognized mandis within the notified period can benefit from it.

PPSS - In the case of oilseeds, States will have the option to roll out PPSSs in select districts. Under this,

a private player can procure crops at MSP when market prices drop below MSP. The private player will

then be compensated through a service charge up to a maximum of 15% of the MSP.

2.26 Ministry of Health and Family Welfare

2.26.1 Nikshay Poshan Abhiyan

It is a direct benefit transfer (DBT) scheme for nutritional support to Tuberculosis (TB) patients

rolled out in April 2018 by Ministry of Health and Family Welfare.

The scheme is a centrally sponsored scheme under National Health Mission (NHM). Financial

norms of NHM in terms of cost sharing are applicable to the scheme. The scheme is not related to

Poshan Mission which is an initiative of Ministry of Women and Child Development.

Page 32: PRELIMS Academy for Civil Services PRESS · 2.13.1 Mission RakshaGyan Shakti 2.14 National Food Security Act (NFSA) 2.15 Ministry of Rural Development 2.15.1 Mission Antyodaya 2.15.2

https://t.me/joinchat/AAAAAFYrI5kpQsEAKqmo-A

Page | 31

This scheme is implemented across all States and UTs in India. All TB patients notified on or after

1st April 2018 including all existing TB patients under treatment are eligible to receive incentives.

The patient must be registered\notified on the NIKSHAY portal. Financial incentive of Rs.500/-

through Direct Benefit Transfer per month for each notified TB patient for duration during which

the patient is on anti-TB treatment.

2.26.2 Pradhan Mantri Swasthya Suraksha Yojana (PMSSY)

It was launched by Ministry of Health & Family Welfare in 2003 with an objective to correct regional

imbalances in the availability of affordable/ reliable tertiary healthcare services and also to augment

facilities for quality medical education in the country.

It has two components

1. Setting up of AIIMS like institutions and

2. Upgrading of Government medical college institutions.

2.26.3 Ayushman Bharat Health Scheme

The National Health Protection Mission or Ayushman Bharat Yojana, launched by the Government is the

first major step which aims to create a healthy, capable and content new India. It will also focus on the

poor and weaker sections of the society. It aims to provide insurance of up to 5 lakh rupees to each

family. The new scheme also intends to improve secondary and tertiary healthcare services for crores of

Indians.

There are two flagship initiatives under Ayushman Bharat:

PradhanMantri Jan ArogyaYojana (PMJAY):Rs. 1200 crore have been allocated for this flagship

programme. Under this scheme, 1.5 lakh health sub-centres are being converted into health and wellness

centres. The scheme will cover more than 10 crore poor families, which is approximately 50 crore

persons. The centres will provide comprehensive healthcare, including treatment for non-communicable

diseases and maternal and child health services. It will also setup wellness centres which will give poor

people OPD facility near their homes.

National Health Protection Scheme: The National Health Protection Scheme will cover over 10 crore

poor and vulnerable families. It will provide coverage up to 5 lakh rupees per family, per year for

secondary and tertiary care hospitalization.

2.26.4 Laqshya Programme

Page 33: PRELIMS Academy for Civil Services PRESS · 2.13.1 Mission RakshaGyan Shakti 2.14 National Food Security Act (NFSA) 2.15 Ministry of Rural Development 2.15.1 Mission Antyodaya 2.15.2

https://t.me/joinchat/AAAAAFYrI5kpQsEAKqmo-A

Page | 32

It was launched by Ministry of Health and Family Welfare and is aimed at improving quality of care in

labour room and maternity Operation Theatre (OT). The Laqshya program is being implemented at all

Medical College Hospitals, District Hospitals and First Referral Unit (FRU), and Community Health Center

(CHCs) and will benefit every pregnant woman and new-born delivering in public health institutions. The

Program aims at implementing „fast-track‟ interventions for achieving tangible results within 18 months.

Under the initiative, a multi-pronged strategy has been adopted such as improving infrastructure up-

gradation, ensuring availability of essential equipment, providing adequate human resources, capacity

building of health care workers and improving quality processes in the labour room. Facilities scoring

more than 90%, 80% and 70% will be given Platinum, Gold and Silver badge accordingly.

2.26.5 Pradhan Mantri Surakshit Matritva Abhiyan (PMSMA)

Implemented by Ministry of Health and Family Welfare, it aims to provide free health check-ups to

pregnant women at government health centers and hospitals.

Program:

The national programme aims to provide pregnant ladies free ante-natal services (ANC) and

required treatment for free on 9th of every month.

Objectives of the scheme are

1. Provide healthy life to the pregnant women.

2. Lowering the maternity mortality rate.

3. Making pregnant women aware of their health issues and diseases.

4. Making sure safe delivery and healthy life of the baby.

The scheme is applicable only for the pregnant women in their pregnancy period of 3 to 6

months.

It will provide all kinds of medical check-ups completely free to pregnant women.

These check-ups will take place at the medical centres, government and private hospitals and

private clinics across the country.

Women will be marked differently using different colour stickers based on their health problems

so that doctors can easily detect the problem.

Different colour stickers will be Red Sticker for Serious patients, Blue Sticker for High blood

pressure and Yellow Sticker for Other diseases.

2.27 Ministry of Agriculture & Farmers Welfare

2.27.1 ENSURE Portal

Page 34: PRELIMS Academy for Civil Services PRESS · 2.13.1 Mission RakshaGyan Shakti 2.14 National Food Security Act (NFSA) 2.15 Ministry of Rural Development 2.15.1 Mission Antyodaya 2.15.2

https://t.me/joinchat/AAAAAFYrI5kpQsEAKqmo-A

Page | 33

The portal “ENSURE”- National Livestock Mission-EDEG is developed by NABARD and is operated under

the Department of Animal Husbandry, Dairying & Fisheries. Under the Mission‟s component called

Entrepreneurship Development and Employment Generation (EDEG), subsidy payment for activities

related to poultry, small ruminants, pigs etc. through Direct Benefit Transfer (DBT) goes directly to the

beneficiary‟s account. In order to make it better, simpler and transparent, the NABARD has developed an

online portal “ENSURE” (https://ensure.nabard.org) so that the information related to beneficiary and

processing of application can be made readily available.

2.27.2 Pradhan Mantri Fasal Bima Yojna (PMFBY)

It was launched by Ministry of Agriculture &

Farmers Welfare with an objective to provide

comprehensive insurance coverage against crop

loss. Launched in 2016, it will replace the

existing two crop insurance schemes National

Agricultural Insurance Scheme (NAIS) and

Modified NAIS

Scheme:

The scheme covers kharif, rabi crops as

well as annual commercial and

horticultural crops. New scheme will

cover post-harvest losses apart from

yield loss.

It will also provide farm level assessment

for localised calamities including

hailstorms, unseasonal rains, landslides

and inundation, pests and diseases.

The scheme proposes mandatory use of

remote sensing, smart phones and

drones for quick estimation of crop loss

to speed up the claim process.

The settlement of claims will be fastened

for the full sum assured. About 25% of

the likely claim will be settled directly on

farmers account. There will not be a cap

on the premium and reduction of the

sum insured.

It also provides insurance benefits to

Landless labourers.

What is new in this scheme?

It is open to all farmers

It is mandatory for farmers availing crop

loans for notified crops in notified areas.

There is no capping in premium and one

premium rate on pan-India basis. It is

1.5%, 2% and 5% for all Rabi, Kharif

and annual horticultural/ commercial

crops, respectively.

The balance premium will be paid by the

government to provide full insured

amount to the farmers. There is no

upper cap on government subsidy, even

if the balance premium is 90 percent,

the government will bear it.

This scheme provides full coverage of

insurance. While NAIS had full coverage,

it was capped in the modified-NAIS

scheme.

It also covers the localized risks such as

hailstorm, landslide, inundation etc.

Earlier schemes did not cover

inundation.

It provides post-harvest coverage. The

NAIS did not cover while the modified

NAIS covered only coastal regions.

A cluster approach will be adopted

under which a group of districts with

Page 35: PRELIMS Academy for Civil Services PRESS · 2.13.1 Mission RakshaGyan Shakti 2.14 National Food Security Act (NFSA) 2.15 Ministry of Rural Development 2.15.1 Mission Antyodaya 2.15.2

https://t.me/joinchat/AAAAAFYrI5kpQsEAKqmo-A

Page | 34

variable risk profile will be allotted to an

insurance company.

Union Government has decided to cover

damages to crops in wild animal attacks

under Pradhan Mantri Fasal Bima

Yojana in select districts on an

experimental basis.

.

2.27.3 Krishonnati Yojana

The Umbrella scheme comprises of 11

Schemes/Missions. These schemes look to

develop the agriculture and allied sector in a

holistic and scientific manner to increase the

income of farmers by enhancing production,

productivity and better returns on produce.

The Schemes that are part of the Umbrella

Schemes are

(i) Mission for Integrated Development of

Horticulture (MIDH) with a total central share of

Rs. 7533.04 crore, to promote holistic growth of

horticulture sector; to enhance horticulture

production, improve nutritional security and

income support tofarm Households.

(ii) National Food Security Mission (NFSM),

including National Mission on Oil Seeds and Oil

Palm (NMOOP), with a total central share of

Rs.6893.38 crore. It aims to increase

production of rice, wheat, pulses, coarse cereals

and commercial crops, through area expansion

and productivity enhancement in a suitable

manner in the identified districts of the country,

restoring soil fertility and productivity at the

individual farm level and enhancing farm level

economy.

(iii) National Mission for Sustainable Agriculture

(NMSA) with a total central share of Rs.3980.82

crore. NMSA aims at promoting sustainable

agriculture practices best suitable to the specific

agro-ecology focusing on integrated farming,

appropriate soil health management and

synergizing resource conservation technology.

(iv)Sub-mission on Agriculture Extension

(SMAE) with a total central share of Rs.2961.26

crore. SMAE aims to strengthen the ongoing

extension mechanism of State Governments,

local bodies etc., achieving food and nutritional

security and socio-economic empowerment of

farmers, to institutionalize programme planning

and implementation mechanism, to forge

effective linkages and synergy amongst various

stake-holders, to support HRD interventions, to

promote pervasive and innovative use of

electronic / print media, inter-personal

communication and ICT tools, etc.

(v) Sub-Mission on Seeds and Planting Material

(SMSP) with a total central share of Rs.920.6

crore. SMSP aims to increase production of

certified / quality seed, to increase SRR, to

upgrade the quality of farm saved seeds, to

strengthen the seed multiplication chain, to

promote new technologies and methodologies in

seed production, processing, testing etc., to

strengthen and modernizing infrastructure for

seed production, storage, certification and

quality etc.

Page 36: PRELIMS Academy for Civil Services PRESS · 2.13.1 Mission RakshaGyan Shakti 2.14 National Food Security Act (NFSA) 2.15 Ministry of Rural Development 2.15.1 Mission Antyodaya 2.15.2

https://t.me/joinchat/AAAAAFYrI5kpQsEAKqmo-A

Page | 35

(vi)Sub-Mission on Agricultural Mechanisation

(SMAM) with a total central share of Rs.3250

crore. SMAM aims to increase the reach of farm

mechanization to small and marginal farmers

and to the regions where availability of farm

power is low, to promote „Custom Hiring Centres‟

to offset the adverse economies of scale arising

due to small landholding and high cost of

individual ownership, to create hubs for hi-tech

and high value farm equipment, to create

awareness among stakeholders through

demonstration and capacity building activities,

and to ensure performance testing and

certification at designated testing centers located

all over the country.

(vii)Sub Mission on Plant Protection and Plan

Quarantine (SMPPQ) with a total central share of

Rs.1022.67 crore. SMPPQ aims to minimize loss

to quality and yield of agricultural crops from

the ravages of insect pests, diseases, weeds,

nematodes, rodents, etc. and to shield our

agricultural bio-security from the incursions and

spread of alien species, to facilitate exports of

Indian agricultural commodities to global

markets, and to promote good agricultural

practices, particularly with respect to plant

protection strategies and strategies.

(viii) Integrated Scheme on Agriculture Census,

Economics and Statistics (ISACES) with a total

central share of Rs. 730.58 crore. It aims to

undertake the agriculture census, study of the

cost of cultivation of principal crops, to

undertake research studies on agro-economic

problems of the country, to fund

conferences/workshops and seminars involving

eminent economists, agricultural scientists,

experts and to bring out papers to conduct short

term studies, to improve agricultural statistics

methodology and to create a hierarchical

information system on crop condition and crop

production from sowing to harvest.

(ix) Integrated Scheme on Agricultural

Cooperation (ISAC) with a total central share of

Rs. 1902.636 crore. It aims to provide financial

assistance for improving the economic

conditions of cooperatives, remove regional

imbalances and to speed up - cooperative

development in agricultural marketing,

processing, storage, computerization and weaker

section programmes; to help cotton growers

fetch remunerative price for their produce

through value addition besides ensuring supply

of quality yarn at reasonable rates to the

decentralized weavers.

(x) Integrated Scheme on Agricultural Marketing

(ISAM) with a total centralshare of 3863.93

crore. ISAM aims to develop agricultural

marketing infrastructure; to promote innovative

and latest technologies and competitive

alternatives in agriculture marketing

infrastructure; to provide infrastructure facilities

for grading, standardization and quality

certification of agricultural produce; to establish

a nationwide marketing information network; to

integrate markets through a common online

market platform to facilitate pan-India trade in

agricultural commodities, etc.

(xi) National e-Governance Plan (NeGP-A) with a

total central share of 211.06 crore aims to bring

farmer centricity & service orientation to the

programmes; to enhance reach & impact of

extension services; to improve access of farmers

to information & services throughout crop-cycle;

to build upon, enhance & integrate the existing

ICT initiatives of Centre and States; and to

enhance efficiency & effectiveness of programs

Page 37: PRELIMS Academy for Civil Services PRESS · 2.13.1 Mission RakshaGyan Shakti 2.14 National Food Security Act (NFSA) 2.15 Ministry of Rural Development 2.15.1 Mission Antyodaya 2.15.2

https://t.me/joinchat/AAAAAFYrI5kpQsEAKqmo-A

Page | 36

through making available timely and relevant

information to the farmers for increasing their

agriculture productivity.

2.27.4 ParamparagatKrishiVikasYojana

The Paramparagat Krishi Vikas Yojana (PKVY) –

traditional farming improvement programme

was launched in 2015. It is an extended

component of Soil Health Management (SHM)

under the Centrally Sponsored Scheme (CSS),

National Mission on Sustainable Agriculture

(NMSA). PKVY aims at supporting and promoting

organic farming, reduction in dependence on

fertilizers and agricultural chemicals, in turn,

resulting in improvement of the soil health while

increasing the yields. Organic food, thus

produced will be linked with modern marketing

tools and local markets.

The revamped PKVY promotes organic farming

through the adoption of organic village by

cluster approach and Participatory Guarantee

System of certification.

Funding pattern:

Funding pattern under the scheme is in the ratio

of 60:40 by the Central and State Governments

respectively. In case of North Eastern and the

Himalayan States, Central Assistance is

provided in the ratio of 90:10 (Centre: State) and

for Union Territories, the assistance is 100%.

Objectives:

The objective is to produce

agricultural products free from

chemicals and pesticides residues

by adopting eco- friendly, low- cost

technologies. Key Thrust areas of

PKVY in promoting organic farming

include the following:

Promote organic farming among

rural youth/ farmers/ consumers/

traders

Disseminate latest technologies in

organic farming

Utilize the services of experts from

public agricultural research system

in India

Organize a minimum of one cluster

demonstration in a village

Implementation of the programme:

PKVY is being implemented by the

Organic Farming cell of the Integrated

Nutrient Management (Division) of

Department of Agriculture; Cooperation

and Farmers Welfare (DAC&FW).

At the State level, State Department of

Agriculture and Cooperation has been

implementing the scheme with the

involvement of Regional Councils that

are registered under PGS- India

Certification Programme.

At the district level, the Regional

Councils (RCs) within the district anchor

the implementation of PKVY.

.

Page 38: PRELIMS Academy for Civil Services PRESS · 2.13.1 Mission RakshaGyan Shakti 2.14 National Food Security Act (NFSA) 2.15 Ministry of Rural Development 2.15.1 Mission Antyodaya 2.15.2

https://t.me/joinchat/AAAAAFYrI5kpQsEAKqmo-A

Page | 37

2.27.5 National Bamboo Mission

National Bamboo Mission is a Centrally Sponsored Scheme started in the year 2006-07. The scheme was

later subsumed under the Mission for Integrated Development of Horticulture (MIDH) for the years 2014-

15 and 2015-16. The scheme aims at promoting the growth of Bamboo sector through the area based

regionally differentiated strategy. The funding pattern will be 60:40 between Centre and State Govt. for all

States except NE & Hilly states, where it would be 90:10 and 100% in case of Union Territories

Objectives:

Enhance the yields with improved and new varieties.

In the potential areas, increase coverage area for bamboo.

Promotion and marketing of bamboo and handicrafts made of bamboo.

Establish coordination and cooperation among stakeholders for the development of the bamboo

sector.

Promote, develop and disseminate technologies through a perfect blend of the modern scientific

knowledge and the traditional wisdom.

Generate employment opportunities for skilled and unskilled labourers, especially unemployed

youths.

The NBM has now been extended till 2019-20. This extended mission aims at ensuring holistic

development of the Bamboo Sector. It aims at establishing effective linkage between the industry and the

producer.

2.27.6 Pradhan Mantri Krishi Sinchayee Yojana(PMKSY)

PMKSY is a Centrally Sponsored Scheme (Core Scheme) launched in 2015. Centre- States will be

75:25 per cent. In the case of the north-eastern region and hilly states, it will be 90:10.

Its objectives are:

o To expand the cultivable area under assured irrigation (HarKhetkopani),

o To improve on-farm water use efficiency to reduce wastage of water,

o To enhance the adoption of precision-irrigation and other water saving technologies (More

crop per drop),

o To enhance recharge of aquifers and introduce sustainable water conservation practices

by exploring the feasibility of reusing treated municipal based water for peri-urban

agriculture and attract greater private investment in a precision irrigation system.

Page 39: PRELIMS Academy for Civil Services PRESS · 2.13.1 Mission RakshaGyan Shakti 2.14 National Food Security Act (NFSA) 2.15 Ministry of Rural Development 2.15.1 Mission Antyodaya 2.15.2

https://t.me/joinchat/AAAAAFYrI5kpQsEAKqmo-A

Page | 38

Components

1. Accelerated Irrigation Benefit Program (AIBP): To concentrate on speedier fulfillment of

continuous Major and Medium Irrigation including National Projects.

2. Har Khet Ko Pani (HKKP): Creation of new water sources through minor water system (both

surface and groundwater); repair, reclamation and redesign of conventional water bodies; charge

range advancement; fortifying and production of dispersion organize from sources to the ranch

and so on.

3. Per Drop More Crop (PDMC): Precision water system frameworks, effective water transport and

application, small scale level stockpiling structures, beating up of info cost past Mahatma Gandhi

National Rural Employment Guarantee Scheme (MGNREGS) admissible cutoff points, auxiliary

stockpiling, water lifting gadgets, augmentation exercises, coordination and administration and

so forth.

4. Watershed Development (WD): Ridge territory treatment, seepage line treatment, soil and

dampness protection, water reaping and other watershed intercessions.

It is formulated by amalgamating ongoing schemes:

1. Accelerated Irrigation Benefit Programme (AIBP) -Ministry of Water Resources, River

Development & Ganga Rejuvenation.

2. Integrated Watershed Management Programme (IWMP) - Department of Land Resources,

Ministry of Rural Development.

3. On-Farm Water Management (OFWM) - Department of Agriculture and Cooperation.

2.28 Union Ministry of Housing and Urban Affairs

2.28.1 PAiSA Portal

Union Ministry of Housing and Urban Affairs launched „PAiSA‟ – Portal for Affordable Credit and Interest

Subvention Access during National Workshop on Municipal Finance and Urban Planning held in New

Delhi.

It is centralised electronic platform for quicker processing of loans under the Deendayal Antyodaya Yojana

– National Urban Livelihoods Mission (DAY-NULM). It aims to connect directly with beneficiaries and

ensure there is greater efficiency in the delivery of services. It has been designed and developed by

Allahabad Bank, the nodal bank under the scheme.

2.29 Ministry of Housing and Urban Poverty Alleviation

Page 40: PRELIMS Academy for Civil Services PRESS · 2.13.1 Mission RakshaGyan Shakti 2.14 National Food Security Act (NFSA) 2.15 Ministry of Rural Development 2.15.1 Mission Antyodaya 2.15.2

https://t.me/joinchat/AAAAAFYrI5kpQsEAKqmo-A

Page | 39

2.29.1 Pradhan Mantri Awas Yojana (Urban)

The Pradhan Mantri Awas Yojana (Urban) Programme launched by the Ministry of Housing and Urban

Poverty Alleviation (MoHUPA), in Mission mode aims at providing affordable housing to the urban poor.

Target

The mission aims at providing Housing for All by 2022 when the Nation completes 75 years of its

Independence.

Under PMAY, it is proposed to build two crore houses for urban poor including Economically

Weaker Sections (EWS), Low Income Groups (LIG) and Middle Income Groups (MIGs) in urban

areas by the year 2022 through a financial assistance from central government.

Components

The Mission seeks to address the housing requirement of urban poor including slum dwellers through

following programme verticals:

Slum rehabilitation of Slum Dwellers with participation of private developers using land as a

resource.

Promotion of Affordable Housing for weaker section through credit linked subsidy.

Affordable Housing in Partnership with Public & Private sectors.

Subsidy for beneficiary-led individual house construction or enhancement.

Targeted group of beneficiaries

Indian women of all religions and castes. Everyone will be equally eligible no biased treatment on

basis of caste or religion.

People who come from low income and economically weaker sections of the society.

SC (Scheduled Castes) and ST (Scheduled Tribes).

Eligibility criteria

The mission seeks to address the housing requirement of urban poor including slum dwellers.

Beneficiaries include economically weaker section (EWS), low-income groups (LIGs) and Middle

Income Groups (MIGs).

A beneficiary family will comprise husband, wife, unmarried sons and/or unmarried daughters.

The beneficiary family should not own a pucca house either in his/her name or in the name of

any member of his/her family in any part of India.

Page 41: PRELIMS Academy for Civil Services PRESS · 2.13.1 Mission RakshaGyan Shakti 2.14 National Food Security Act (NFSA) 2.15 Ministry of Rural Development 2.15.1 Mission Antyodaya 2.15.2

https://t.me/joinchat/AAAAAFYrI5kpQsEAKqmo-A

Page | 40

2.30 Important State Government schemes:

2.30.1 Nirman Kusuma Programme

Odisha Government on October 6, 2018 launched 'Nirman Kusuma' programme to provide financial

assistance to the children of the construction workers for their technical education in the state.

The scheme will help fulfill the dreams of the construction workers' children. The scheme would provide

financial assistance to the children of construction workers for their education in Industrial Training

Institutes (ITIs) and polytechnics.

Key Highlights of the Programme

Under the programme, an ITI student will be entitled to get financial assistance worth Rs. 23,600

and a diploma student will get assistance worth Rs. 26,300 per annum.

In total, around 1878 students are expected to be benefited from the programme.

The government has also doubled the compensation given in case of an accident of a construction

worker from Rs. 2 lakh to Rs. 4 lakh.

2.30.2 Sambal Scheme

Madhya Pradesh Government has Sambal Yojana, an outstanding power bill waiver scheme and

subsidised power scheme for labourers and poor families. Under this scheme, Below Poverty Line (BPL)

families and registered labourers of unorganised sector from the state will be provided electricity at cost of

Rs. 200 per month.

2.30.3 Rythu Bandhu

Why in news?

Telangana‟s Rythu Bandhu scheme supports its farmers with Rs. 4,000 for every acre they own.

What is the Rythu Bandhu Scheme about?

Rythu Bandhu is a support scheme for farmers in Telangana, which provides cheque payments to

farmers based on their landholdings

The objective is to help the farmer meet a major part of his expenses on seed, fertiliser, pesticide,

and field preparation.

Page 42: PRELIMS Academy for Civil Services PRESS · 2.13.1 Mission RakshaGyan Shakti 2.14 National Food Security Act (NFSA) 2.15 Ministry of Rural Development 2.15.1 Mission Antyodaya 2.15.2

https://t.me/joinchat/AAAAAFYrI5kpQsEAKqmo-A

Page | 41

The scheme covers 1.42 crore acres in the 31 districts of the state, and every farmer owning land

is eligible.

The government will issue cheques rather than make direct benefit transfer (DBT) because banks

might use the DBT money to adjust against farmers‟ previous dues.

The government plans to extend the flat Rs. 4,000-per-acre subsidy to the Rabi season as well,

with distribution of cheques.

What are few limitations of the scheme?

The Rythu Bandhu scheme does not exclude rich farmers and wealthy landlords.

The scheme leaves out tenant cultivators an estimated 40% of Telangana‟s farming population

and mostly coming from the poorest and most disadvantaged backgrounds.

Tenant farmers cannot be included in the scheme as they cannot submit any proof of cultivation

of land, which is done mostly based on informal and oral lease arrangements.

2.30.4 Swayangsiddha Initiative

Swayangsiddha, which means self-reliance, will be executed by the West Bengal Police.

The scheme aims to empower young boys and girls to make informed choices so that they are less

vulnerable to trafficking and child marriage.

Swayangsiddha Groups have been formed in schools and colleges with interested students.

These groups were formed with students between the age group of 12 to 21 years.

Particulars of the Mission-

Raising awareness on human, gender and child rights and strengthening prevention of human

trafficking and child marriage using a converging approach.

Engaging youth from different schools and colleges to combat human trafficking and child

marriage

Strengthening response mechanism in collaboration with Police and child protection committees

to build safe community

Strengthening access to schemes and entitlements on education, training, livelihood and food

security for vulnerable groups.

2.30.5 Mukhyamantri Yuva Nestham

The AP government has launched this scheme through which an allowance of Rs. 1000 per month will be

provided to unemployed youth in the state. About 12 lakh youths in the age group of 22-35 years will get

the benefit of the scheme. The scheme will be extended to all those eligible even if there is more than one

Page 43: PRELIMS Academy for Civil Services PRESS · 2.13.1 Mission RakshaGyan Shakti 2.14 National Food Security Act (NFSA) 2.15 Ministry of Rural Development 2.15.1 Mission Antyodaya 2.15.2

https://t.me/joinchat/AAAAAFYrI5kpQsEAKqmo-A

Page | 42

beneficiary in a family. The money will be credited directly into the bank accounts through biometric

authentication.

2.30.6 Kanyashree Scheme

It is conditional cash transfer scheme aimed at improving status and well-being of girl child by

incentivising schooling of teenage girls and delaying their marriages until the age of 18. It was launched

by West Bengal Government (Women Development and Social Welfare department) in October 2013. It

was aimed at increasing educational attainment of girls, prevention of child marriage and financial

inclusion.

Objectives:

Improve lives and status of the adolescent girls in State.

Provide financial help to girls from disadvantaged families to pursue higher studies (now it will

cover every girl from state).

Prevent child marriage.

Improved outcomes in terms of their health (especially facilitate the prevention of infant and

maternal mortality).

Contribute towards empowerment of girls in the state.

Bring immeasurable benefits for the larger society as a whole.

Targeted beneficiaries: All girl children within age of 13 to 19 in the state. Girls regularly attending

institutions for education or vocational or sports training. Girls of Child Care Institutes registered under

Juvenile Justice Act (JJ), 2000 within age of 18-19 years.

2.30.7 Krushak Assistance for Livelihood and Income Augmentation

(KALIA) scheme

Krushak Assistance for Livelihood and Income Augmentation (KALIA) is a support scheme of Odisha

whose primary targets are small farmers, cultivators and landless agricultural labourers. The scheme

involves payments to encourage cultivation and associated activities.

The important features of the scheme are:

Under the Scheme, Odisha would spend Rs. 10,180 crore over three years until 2020-21 in

providing financial assistance to cultivators and landless agricultural labourers benefitting 92%

of the cultivators in the state and including every category from big farmers to landless

cultivators. The government would provide Rs. 10,000 per family as assistance for cultivation,

Rs.5,000 each in the Kharif and Rabi seasons, for five cropping seasons between 2018-19 and

2021-22.

Page 44: PRELIMS Academy for Civil Services PRESS · 2.13.1 Mission RakshaGyan Shakti 2.14 National Food Security Act (NFSA) 2.15 Ministry of Rural Development 2.15.1 Mission Antyodaya 2.15.2

https://t.me/joinchat/AAAAAFYrI5kpQsEAKqmo-A

Page | 43

The scheme also targets landless households, specifically SC and ST families. They will be

supported with a unit cost of Rs. 12,500 for activities like oat rearing, mushroom cultivation,

beekeeping, poultry farming and fishery. The Scheme also provides a life insurance cover of Rs. 2

lakh and addition additional personal accident coverage of the same amount for 57 lakh

households.

2.30.8 Suryashakti Kisan Yojana (SKY) Scheme

With an aim to generate additional income for farmers, the Gujarat Government launched Suryashakti

Kisan Yojna (SKY) under which grid-connected solar panels will be provided to over 12,000 cultivators

who have already taken regular electricity connections for irrigation purpose.

The scheme aims at empowering farmers of state to generate their own electricity using solar energy and

help doubling their income. Under it, farmers of state having existing electricity connection will be given

solar panels as per their load requirements. It envisages setting up of separate feeders for agricultural

solar energy consumption.

Cost Sharing: The State and Central governments will give 60% subsidy on cost of project. The farmer is

required to take 5% cost, while 35% will be provided to him as affordable loan with interest rates of 4.5 to

6%.

2.31 Others

2.31.1 Ujjwala Sanitary Napkins Initiative

Ujjwala Sanitary Napkins initiative has been launched by three oil marketing companies – IOCL, BPCL

and HPCL.

Key highlights of the scheme:

The mission, which forms part of the CSR initiative of OMCs in Odisha, is aimed to educate

women on female hygiene and health, improve accessibility to low cost eco-friendly sanitary pads

and boost rural employment and economy.

The three companies will set up 100 manufacturing units at the Common Service Centres (CSC)

covering 93 Blocks across 30 districts of Odisha at an estimated cost of Rs. 2.94 crore.

At least 10 Ujjwala beneficiary women will get employment at each CSC. Each facility will have a

capacity to produce 1,200-2,000 pads per day and will have a sterilisation room to ensure that

the napkins are sterilised before they are packed for use by rural women.

2.32 Ashok Dalwai Committee: Doubling Agricultural Income by 2022

Page 45: PRELIMS Academy for Civil Services PRESS · 2.13.1 Mission RakshaGyan Shakti 2.14 National Food Security Act (NFSA) 2.15 Ministry of Rural Development 2.15.1 Mission Antyodaya 2.15.2

https://t.me/joinchat/AAAAAFYrI5kpQsEAKqmo-A

Page | 44

Important recommendations:

One-India market

The one-India market concept may

benefit from placing agricultural

marketing under the Concurrent List (in

the Seventh Schedule of the

Constitution).

The needs include creation of better

physical infrastructure, improved price

information dissemination campaigns,

and reform regulations that force

farmers to sell their produce to local

monopolies.

Electronic National Agriculture Market (NAM)

Role of FPO/VPO:

It suggested that farmer producer and

village producer organisations

(FPO/VPO) could play a critical role in

integrating small and marginal farmers

into the agricultural market system.

The report set a minimum target of

7,000 FPOs/VPOs, each of which could

cover 1,000 farmers and/or 1,000

hectares.

The committee also called for amending

the Companies Act to facilitate private

sector shareholding in FPOs up to 26

per cent and incentivising them by

treating them at par with cooperative

societies.

Setting up Markets:

The committee estimated that the

country would need about 10,000

wholesale and nearly 20,000 rural retail

markets to achieve the desired market

density to build a pan-India system.

The current agricultural marketing

system in the country comprised of

2,284 Agricultural Produce marketing

Committees (APMCs), which operate

2,339 principal markets. These principal

markets have extended their footprint

further through sub-market yards,

numbering 4,276.

State Governments may convert these

principal and sub-market yards into

full-fledged and independent markets.

This will take the total number of

wholesale markets to more than 6,600

and the remaining requirement of about

3,500 may be met by promoting private

markets under the provisions of the

proposed Agricultural Produce and

Livestock Marketing, (Promotion and

Facilitation) Act, 2017.

APLM rollout sought

The committee also urged the Union

Agriculture Ministry to roll out the

Model Agricultural Produce and

Livestock Marketing (Promotion and

Facilitating) Act (APLM) Rules so that

States can make the act operational.

States could upgrade existing facilities

such as warehouses and silos as

markets.

The demand for rural retail markets

could be met by upgrading the existing

over-20,000 rural periodical markets as

Primary Rural Agricultural Markets.

It also delineated the need for both the

Centre as well as the States/UTs

constituting special purpose vehicles to

Page 46: PRELIMS Academy for Civil Services PRESS · 2.13.1 Mission RakshaGyan Shakti 2.14 National Food Security Act (NFSA) 2.15 Ministry of Rural Development 2.15.1 Mission Antyodaya 2.15.2

https://t.me/joinchat/AAAAAFYrI5kpQsEAKqmo-A

Page | 45

own and operate the National

Agriculture Market.

The Ministry could seek the help of an

expert for specialist advice on the

transactions involved.

Financial support:

The committee believes that small and

marginal farmers, who constitute 80 per

cent of Indian farmers, would benefit

from an efficient marketing system, only

if they have withholding capacity.

This can be achieved by offering them

pledge finance (post-harvest loan

against produce as collateral).

Storage godowns, including cold

storages, should be upgraded per the

standards laid down by the

Warehousing Development and

Regulatory Authority so that they can

issue Negotiable Warehouse Receipts.

The Ministry has to develop

comprehensive guidelines to promote

warehouse-based post-harvest loans

and eNWR based trading.

There is also a need to popularise post-

harvest loans against NWRs among

farmers and orient financial institutions

to participate in the pledge loan system.

2.33 Healthy States Progressive India Report

Healthy States, Progressive India Report is a

comprehensive Health report released by the

National Institution for Transforming India (NITI

Aayog). It is an annual report which would rank

States and Union Territories on incremental

changes in health outcomes and overall

performance with respect to the others. It has

been developed by NITI Aayog in consultation

with Technical Assistance Agency of World

Bank, Ministry of Health and Family Welfare

(MoHFW), States and Union Territories, domestic

and international sector experts and other

development partners.

The Health Index is based on three main

domains that lays its focus on health outcomes,

governance and information, and critical inputs–

Health Outcomes

10 indicators weighing 70% of the total index

score has been given the highest importance.

Key indicators under this domain are Neonatal

Mortality, Under Five Mortality Rate, Total

Fertility Rate, and Sex Ratio at Birth etc.

The objective of the index

To develop a composite health index

based on key health outcomes, health

system and service delivery indicators.

To ensure State and UTs partnership

and ownership through Health Index

data submission on web-based portals.

Build transparency through

independent validation by independent

agencies.

Generate Health Index scores and

ranking of States and UTs based on year

to year overall performance.

Governance and Information

3 indicators which weigh 12% are Data integrity

measure, the average occupancy of an officer

Page 47: PRELIMS Academy for Civil Services PRESS · 2.13.1 Mission RakshaGyan Shakti 2.14 National Food Security Act (NFSA) 2.15 Ministry of Rural Development 2.15.1 Mission Antyodaya 2.15.2

https://t.me/joinchat/AAAAAFYrI5kpQsEAKqmo-A

Page | 46

(signifying stability in the organization) and

average occupancy of a full-time officer for all

districts. Key Inputs and Processes – 10

indicators weighing 18% are the proportion of

vacant healthcare, the proportion of total staff

for which an e-payslip can be generated etc.

The states have been categorised based

on the availability of data and the fact

that similar states should be compared

amongst themselves. Therefore the

states have been categorised as larger

states, smaller States and UTs.

Based on the above categories the states

are grouped into three categories –

Aspirants (bottom one-third states with

a score below 48), Achievers (middle one

third state with a score between 48 and

63) and Frontrunners with scores above

63.

The composite score of the index is

calculated for a base year i.e. 2014-15

and a reference year i.e. 2016-16. The

incremental ranks are a measure of the

difference in performance between these

two periods

The data sources for the index are

Sample Registration System, Health

Management Information System,

Central MoHFW data, State Report,

National Family Health Survey, Civil

Registration System etc.

Q.1. With reference to Saubhagya scheme, consider the following statement:

1. It was launched in September 2017 by Ministry of Rural development

2. It is being funded to extent of 60% by central grants, 30% by bank loans and 10% by states

3. Under it, free electricity connections are provided to below poverty line (BPL) households, while other households have to pay 300 for the connection.

Which of the following is/are correct? a. 2 only b. 2 and 3 c. 1 and 3 d. 1, 2 and 3 Answer: a Q.2. Which of the following statements about AB-NHPM (Ayushman Bharat –National Health Mission) is/are correct? 1.It will be launched as a Central Sector Scheme 2.All pre-existing conditions will be covered 3.It will subsume only one on-going scheme –the RashtriyaSwasthyaBimaYojana Which of the following is/are correct? a. 1 only b. 2 and 3 only c. 2 only d. 1, 2 and 3 Answer: c

Q.3. Which of the following schemes/initiatives is/are a part of the DeendayalAntyodayaYojana –National Rural Livelihoods Mission (DAY-NRLM)? 1.DeenDayalUpadhyaya –

GrameenKaushalyaYojana (DDU-GKY) 2.DDU Gram JyotiYojana (DDUGJY) 3.MahilaKisanSahshaktikaranPariyojana (MKSP) 4.Start Up Village Entrepreneurship Programme (SVEP) Which of the following is/are correct? a. 1, 3 and 4 only b. 1 and 2 only c. 3 only d. 1, 2, 3 and 4 Answer: a Q.4. With reference to Members of Parliament Local Area Development Scheme(MPLADS), consider the following statements:

1. It was launched in December, 1993 by

Ministry of Parliamentary affairs 2. MPLADS is a centrally-sponsored plan

scheme fully funded by the government of India

3. The funds released under the scheme are non-lapsable.

4. Nominated members of the LokSabha may select works for implementation anywhere in the state they are nominated from.

Which of the following is/are correct? a. 1 and 4

Page 48: PRELIMS Academy for Civil Services PRESS · 2.13.1 Mission RakshaGyan Shakti 2.14 National Food Security Act (NFSA) 2.15 Ministry of Rural Development 2.15.1 Mission Antyodaya 2.15.2

https://t.me/joinchat/AAAAAFYrI5kpQsEAKqmo-A

Page | 47

b. 2, 3 and 4 c. 1, 2, 3 and 4 d. 2 and 3 Answer: d Q.5. The recently announced National Health Protection Scheme is an intervention to address health in which of the following systems?

1. Primary healthcare 2. Secondary healthcare 3. Tertiary healthcare

Which of the following is/are correct? a. 1 and 2 b. 1 and 3 c. 2 and 3

d. 1, 2 and 3

Answer: c Q.6. Consider the following statements about PradhanMantriRojgarProtsahanYojana (PMRPY):

1. The Scheme aims to incentivise employers for employment generation by the Government paying the employers‟ EPS contribution of 8.33%, for the new employees, for the first five years of their employment.

2. The scheme is being implemented by the Ministry of Finance.

3. The Scheme is targeted for employees earning wages less than Rs. 15,000/-per month.

Which of the above statements is/are correct? a. 1 and 2 b. 3 only c. 1, 2 and 3 d. 2 and 3 Answer: b Q.7. With reference to AtalBhujalYojana (ABHY), consider the following statement:

1. It is designed as a Centrally Sponsored Scheme with a total outlay of Rs. 6,000 Crore and is proposed to be implemented with World Bank assistance.

2. The scheme is to be implemented over a

period of five years from 2018-19 to 2022-23.

3. The scheme is launched in states of Gujarat, Haryana, Karnataka, Maharashtra, Uttar Pradesh, Rajasthan and Tamil Nadu.

Which of the following is/are correct? a. 1 and 3 b. 2 and 3 c. 2 only d. 1, 2 and 3 Answer: c

Q.8. Consider the following statement with reference to Ujjwalaprogramme:

1. It was launched by Ministry of Petroleum and Natural Gas

2. Under the scheme 5 Crore LPG (liquefied petroleum gas) connections to poor households will be provided to an adult woman member of BPL family identified through Socio-Economic Caste Census (SECC) data.

3. Tagline of the Programme is SwachhIndhan, BehtarVatavaran.

Which of the following is/are correct? a. 1 only

b. 2 and 3

c. 1 and 2 d. 1 and 3 Answer: a Q.9. With reference to Atal Pension Yojana, consider the following statement

1. It is administered by the Pension Fund Regulatory and Development Authority.

2. The minimum age of joining APY is 18 years and maximum age is 60 years

3. The Central Government would also co-contribute 50% of the subscriber‟s contribution or Rs. 500 per annum, whichever is lower, to each eligible subscriber account, for a period of 5 years.

Which of the following is/are incorrect? a. 1 and 2 b. 2 and 3 c. 1, 2 and 3 d. 1 and 3 Answer: b Q.10. With reference to POSHAN Abhiyan, consider the following statements:

1. POSHAN Abhiyan was launched in Ranchi, Jharkhand in March 2018

2. It is implemented by Ministry of Women and Child Development and aims at

ensuring holistic development and adequate nutrition for pregnant women,

mothers and children Which of the above statements is/are correct? a. 1 only b. 2 only c. Both 1 and 2 d. Neither 1 nor 2 Answer: b

Q.11. BetiBachaoBetiPadhao scheme is implemented by

1. Women and Child Development (WCD)

Page 49: PRELIMS Academy for Civil Services PRESS · 2.13.1 Mission RakshaGyan Shakti 2.14 National Food Security Act (NFSA) 2.15 Ministry of Rural Development 2.15.1 Mission Antyodaya 2.15.2

https://t.me/joinchat/AAAAAFYrI5kpQsEAKqmo-A

Page | 48

2. Health & Family Welfare (MoHFW) 3. Human Resource Development (HRD).

Which of the following is/are correct? a. 1 only b. 2 only c. 3 only d. 1, 2 and 3 Answer: d Q.12. Consider the following statements about the PradhanMantriMatruVandanaYojana (PMMVY), is a Maternity Benefit Programme:

1. It is being implemented in all districts where more than 50 per cent of the

population is recognised as being multi-dimensionally deprived through the Socio Economic Caste Census.

2. A conditional cash incentive of 5,000 rupees in three installments is provided to a pregnant woman.

3. In case of miscarriage or still birth, the beneficiary would be eligible to claim only the remaining installments in the event of any future pregnancy.

Which of the statements given above is/are correct? a. 1 and 2 b. 2 and 3 only c. 1 and 3 only d. 1, 2 and 3 Answer: b Q.13. With reference to SamagraShiksha Scheme, consider the following statement:

1. It is overarching programme for the school education sector extending from pre-school to class X.

2. It subsumes the three erstwhile centrally sponsored schemes i.e. SarvaShikshaAbhiyan (SSA), RashtriyaMadhyamikShikshaAbhiyan (RMSA) and Teacher Education(TE).

Which of the following is/are correct? a. 1 only b. 2 only c. Both 1 and 2 d. Neither 1 nor 2 Answer: b Q.14. The schemes that are part of PradhanMantriAnnadataAaySanrakshanAbhiyan (PM-AASHA) are:

1. Price Support Scheme (PSS), 2. Price Deficiency Payment Scheme

(PDPS) 3. Pilot of Private Procurement and

Stockist Scheme (PPPS) 4. PradhanMantriKrishiSinchaiYojana(PMK

SY)

Which of the following is/are correct? a. 1 and 2 b. 1, 2 and 3 c. 2, 3 and 4 d. 2 and 3 Answer: a

Q.15. Consider the following statements about the progress made by PradhanMantri Mudra Yojana: 1.Shishu loans account for majority of all MUDRA loans. 2.The scheme has been supported by almost all financial sector entities except private sector banks. 3.Over six lakh crore rupees has been disbursed

under the scheme till date. Which of the statements given above is/are correct? a. 1 and 2 only b. 2 and 3 only c. 1 and 3 only d. 1, 2 and 3 Answer: c Q.16. NikshayPoshanAbhiyan is related to a. Maternal health b. Old age health c. Neo-natal Child health d. TB Patients Answer: d Q.17. ThePradhanMantriSwasthyaSurakshaYojana was announced in 2003 with the objectives of

1. Early identification of defects at birth, development delays and early intervention for children

2. Setting up of AIIMS like institutions and upgradation of Government medical college institutions.

Which of the following is/are correct? a. 1 only b. 2 only c. Both 1 and 2

d. Neither 1 nor 2 Answer: b Q.18. The recently launched „LaQshya‟ programme is aimed at a. Improving quality of life in towns and cities b. Reducing maternal mortality rates c. Tripling the railways‟ freight traffic by 2030 d. Elimination of tuberculosis by 2025 Answer: b.

Page 50: PRELIMS Academy for Civil Services PRESS · 2.13.1 Mission RakshaGyan Shakti 2.14 National Food Security Act (NFSA) 2.15 Ministry of Rural Development 2.15.1 Mission Antyodaya 2.15.2

https://t.me/joinchat/AAAAAFYrI5kpQsEAKqmo-A

Page | 49

Q.19. Which of the following are correctly matched? Scheme Ministry

1. PM RojgarProtsahanYojana Ministry of skill development and Entrepreneurship 2. Nutrient based Subsidy Scheme Ministry of Agriculture and Farmers welfare 3. Kusum Ministry of New and Renewable Energy 4. ReUnite Ministry of Commerce and Industry

a. 1 and 2 b. 2, 3 and 4 c. 1 and 4 d. 1, 2, 3 and 4 Answer: c Q.20. Which of the following are correctly matched? Scheme Ministry

1. Satat Ministry of Petroleum and Natural Gas

2. PradhanMantri Jan VikasKaryakram Ministry of Minority Affairs 3. JiyoParsi Scheme Ministry of Culture 4. Project Stree Swabhimaan Ministry of Women and child development

a. 1 and 3 b. 2, 3 and 4 c. 1 and 2 d. 1, 2, 3 and 4 Answer c